Salby Sees Little CO2 Driving Mechanism …Skeptical View Of CO2 Science Is In Fact ‘Textbook Science’

SalbyBy Kenneth Richard

We routinely read from fellow skeptics that they wish Dr. Murry Salby’s research could be made available in written form, or perhaps in a peer-reviewed paper.

Indeed we do have access to his Youtube lecture research (at least a written summary of it) from an even better source than peer-reviewed paper: Dr. Murry Salby’s 2012 university-level textbook: Physics of the Atmosphere and Climate.

Here is a pdf link to the full textbook written by a world-renown expert on atmospheric physics (he’s published several dozen papers in the scientific literature on the subject). We therefore can effectively say that a skeptical view of the CO2-dominated climate paradigm is actually textbook science, not “fringe” science for the “3 percent”.

Below I’ve compiled a short list of some of the written statements from the textbook (emphasis added):

(a) temperature changes occur first and lead to CO2 emission from natural sources (e.g., more ocean outgassing upon warming, more CO2 retention as the ocean cools), indicating that warmer temperatures are driving up CO2 concentrations significantly more than human activity or fossil fuels;

(b) CO2 only accounts for a small portion of the greenhouse effect relative to water vapor/cloud; and

(c) our presumptions about paleoclimate CO2 concentrations are probably inaccurate (too low and too stable), as significant temperature fluctuations would have caused wider fluctuations in CO2 concentrations than current proxy-based reconstructions indicate.

Page 546:

Together, emission from ocean and land sources (∼150 GtC/yr) is two orders of magnitude greater than CO2 emission from combustion of fossil fuel. These natural sources are offset by natural sinks, of comparable strength. However, because they are so much stronger, even a minor imbalance between natural sources and sinks can overshadow the anthropogenic component of CO2 emission.”

And page 249:

The vast majority of that [greenhouse] warming is contributed by water vapor. Together with cloud, it accounts for 98% of the greenhouse effect.” pg. 249

Page 249/50:

Surface temperature depends on the atmosphere’s optical depth. The latter, in turn, depends on atmospheric composition through radiatively active species. Water vapor is produced at ocean surfaces through evaporation. Carbon dioxide is produced by decomposition of of organic matter. These and other processes that control radiatively active species are temperature dependent.”

Page 253:

Revealed by natural perturbations to the Earth-atmosphere system, the sensitivity accounts for much of the observed variation of CO2 emission on interannual time scales (Fig. 1.43). It establishes that GMT cannot increase without simultaneously increasing CO2 emission – from natural sources.”

Page 253:

The results for the two periods are in broad agreement. Together with the strong dependence of CO2 emission on temperature (Fig. 1.43), they imply that a significant portion of the observed increase in r˙CO2 derives from a gradual increase in surface temperature.”

Page 546:

Warming of SST (by any mechanism) will increase the outgassing of CO2 while reducing its absorption. Owing to the magnitude of transfers with the ocean, even a minor increase of SST can lead to increased emission of CO2 that rivals other sources.”

Page 254:

The resemblance between observed changes of CO2 and those anticipated from increased surface temperature also points to a major inconsistency between proxy records of previous climate. Proxy CO2 from the ice core record (Fig 1.13) indicates a sharp increase after the nineteenth century. At earlier times, proxy CO2 becomes amorphous: Nearly homogeneous on time scales shorter than millennial, the ice core record implies virtually no change of atmospheric CO2. According to the above sensitivity, it therefore implies a global-mean climate that is “static,” largely devoid of changes in GMT and CO2. Proxy temperature (Fig. 1.45), on the other hand, exhibits centennial changes of GMT during the last millennium, as large as 0.5–1.0◦ K. In counterpart reconstructions, those changes are even greater (Section 1.6.2). It is noteworthy that, unlike proxy CO2 from the ice core record, proxy temperature in Fig. 1.45 rests on a variety of independent properties. In light of the observed sensitivity, those centennial changes of GMT must be attended by significant changes of CO2 during the last millennium. They reflect a global-mean climate that is “dynamic,” wherein GMT and CO2 change on a wide range of time scales. The two proxies of previous climate are incompatible. They cannot both be correct.”

These statements fully correspond with some of the main themes of his lectures.

173 responses to “Salby Sees Little CO2 Driving Mechanism …Skeptical View Of CO2 Science Is In Fact ‘Textbook Science’”

  1. mwhite

    “The end of the world as we know it? CO2 levels to reach a ‘tipping point’ on 6 June – and Earth may never recover, expert warns”

    http://www.dailymail.co.uk/sciencetech/article-3586669/The-end-world-know-CO2-levels-reach-tipping-point-6-June-marking-milestone-global-warming.html

    We’re all doomed, Dr Paul Fraser

    1. Peter Lester

      OMG – The Athasbasca “Tar” sands is a carbon bomb! The Keystone XL Pipeline would become the artery of death! But actually June 6 deadline for changing course is all wrong. The world will end at midnight tonight…..12:30 in Newfoundland! 😉

    2. DirkH

      “and Earth may never recover, expert warns”

      Must be an expert in lying.
      https://en.wikipedia.org/wiki/Carbon_dioxide_in_Earth's_atmosphere

      1. DirkH
  2. BobW in NC

    Pierre – excellent! Points “a” and “b” before the excerpts are so simple and follow physical principles…how in the “heck” could these not be shoved in the face (sorry – “pointed out”) of warmists? I do not understand.

    Last I heard, water vapor 1) has almost the same LIR absorption as CO2, 2) atmospheric concentration are 1% (poles) to 4% (equator) or approximately 25 to 100 times that of CO2 @ 0.04%, so water vapor by far overrides any significant warming effect of CO2!

    Great post!

    1. Kenneth Richard

      Point “a” has actually been written about for decades. Below are two 1982 papers (and another from 2005) that indicate that temperature changes occur first, then CO2 changes function more like the effect, with slower rates of emissions increases in cool years and higher rates in warm years. These lower and higher emission rates, of course, do not correlate with anthropogenic emissions rates.

      Flohn, 1982
      https://www2.meteo.uni-bonn.de/bibliothek/Flohn_Publikationen/K287-K320_1981-1985/K299.pdf
      The role of the carbon budget has aroused general interest among paleoclimatologists, after Delmas et al., (1980), Berner et al., (1980) and Oeschger (in et al. 1980) found evidence in Antarctic and Greenland ice cores, that the CO2-content of the atmosphere has varied between about 180 ppm during the last glacial (18 ka ago, ka = 103 years) and about 350 ppm (perhaps 400 ppm) during the Holocene warm epoch 6-8 ka ago. Due to the recent wide-spread concern about the climatic consequences of the continuous increase of atmospheric CO2 (Bach, 1980), from about 295 ppm at the end of the 19th century to nearly 340 ppm (1981), the problem of the carbon budget in atmosphere, ocean and biosphere is now thoroughly investigated. … The recent increase of the CO2-content of air varies distinctly from year to year, rather independent from the irregular annual increase of global CO2-production from fossil fuel and cement, which has since 1973 decreased from about 4.5 percent to 2.25 percent per year (Rotty 1981). Comparative investigations (Keeling and Bacastow 1977, Newll et al. 1978, Angell 1981) found a positive correlation between the rate of increase of atmospheric CO2 and the fluctuations of sea surface temperature (SST) in the equatorial Pacific, which are caused by rather abrupt changes between upwelling cool water and downwelling warm water (“El Niño”) in the eastern equatorial Pacific. Indeed the cool upwelling water is not only rich in (anorganic) CO2 but also in nutrients and organisms. (algae) which consume much atmospheric CO2 in organic form, thus reducing the increase in atmospehreic CO2. Conversely the warm water of tropical oceans, with SST near 27°C, is barren, thus leading to a reduction of CO2 uptake by the ocean and greater increase of the CO2. … A crude estimate of these differences is demonstrated by the fact that during the period 1958-1974, the average CO2-increase within five selective years with prevailing cool water only 0.57 ppm/a [per year], while during five years with prevailing warm water it was 1.11 ppm/a. Thus in a a warm water year, more than one Gt (1015 g) carbon is additionally injected into the atmosphere, in comparison to a cold water year.
      —–
      Warren, 1982
      http://www.atmos.washington.edu/~sgw/PAPERS/1982_ice_essay.pdf
      Carbon Dioxide: Other feedbacks are now appearing which are likely to affect the magnitude of glacial-interglacial climatic changes, however these changes are initially forced. The lowered CO2 content of the atmosphere toward the end of the Wisconsin (Neftel et al., 1982) very likely helped cool the climate, and it may well have been the result of the glaciation itself. Broecker (1982) showed how this might happen. Deposition of phosphate-rich biological sediments on shelves when sea level rises, and erosion of these shelves when sea level drops, causes the phosphate content of ocean surface water to be higher during glacial times when sea level is low. The biological productivity of surface water, limited by phosphate, would thus be higher in glacial times and would fix more CO2, reducing the CO2 concentration in surface waters and thus also in the atmosphere. Another mechanism for increasing atmospheric CO2 as sea level rose is that of Berger (1982): coral reef growth precipitates CaCO3, from bicarbonate, thus acidifying the ocean surface water and releasing CO2. … Other forcings which can cause climatic change in a ‘transitive’ climatic system are sustained changes in solar output and changes in atmospheric content (e.g. CO2, volcanic ash). [However, the atmospheric CO2 content, as mentioned above, may not be an ‘external’ cause but may actually be affected by ice sheet sizes. The same may true for the level of volcanic activity, as suggested by Rampino et al. (1979). Gow and Williamson (1971) found much more volcanic ash in the Byrd Station core in ice corresponding to the last glacial maximum. A nearby source was indicated, suggesting that the Antarctic volcanoes were more active at that time, possibly responding to the increase in ice load (Kyle et al., 1981)].
      —-
      Jones and Cox, 2005
      http://onlinelibrary.wiley.com/doi/10.1029/2005GL023027/full
      There is clear similarity between Figures 1b and 1c, with the positive CO2 growth rate anomalies corresponding to El Niño events, and the negative growth rate anomalies corresponding to La Niña events. The largest positive CO2 growth rate anomalies are coincident with large Niño3 values in 1973, 1988 and 1998. … It is unlikely that these anomalies can be explained by an abrupt increase in anthropogenic emissions, as the anomalies are much larger than annual increases in fossil fuel emissions. Most interannual variability in the CO2 growth rate is attributable to variations in land-atmosphere CO2 exchange with climate (e.g., associated with ENSO or volcanic perturbations)

  3. Willis Eschenbach

    “The vast majority of that [greenhouse] warming is contributed by water vapor. Together with cloud, it accounts for 98% of the greenhouse effect.” pg. 249 – “

    Say what? That’s not anywhere near true. Not only that, but he provides no citation for it.

    Go to the MODTRAN site, where you can do the calculations yourself of the amount of radiation absorbed by one, the other, both, and neither::

    Both CO2 and H2O, amount absorbed 98.973
    H2O only, amount absorbed 68.138
    CO2 only, amount absorbed 41.134
    No CO2 or H2O, amount absorbed 0

    Note that the combination of H2O only, plus CO2 only, adds up to more than the amount absorbed by the combination of the two. This is because the absorption bands overlap.

    In other words … as is often the case. Professor Salby is simply making things up and hoping that you’ll believe them.

    w.

    1. Kenneth Richard

      Willis:

      Perhaps you’ll find the below paper by Lightfoot and Mamer (2014), which uses the IPCC’s own figures and two different methods to arrive at the almost identical percentage of the GHE contributed by water vapor (~96%) explanatory and helpful.

      Even Gavin Schmidt has CO2’s proportion to the greenhouse effect at 20%, not 41.134%, which apparently is the “truth” as you see it, meaning that you apparently believe that all other figures for the CO2 GHE contribution percentage that don’t agree with the one from the MODTRAN site must be “made up” lies.

      When you accuse Dr. Salby of “making things up”, Willis, you probably need to substantiate it with something more qualitative than linking to another site’s figures and claiming those figures are right, and therefore Dr. Salby is lying. Just because someone else’s figures are different than the ones you prefer doesn’t mean that others are “making things up.” Is Dr. Lindzen “making things up” because his calculations have ECS at 0.7C (Lindzen and Choi, 2011) while other scientists have ECS at over 3C? Is that really how scientists are supposed to conduct themselves – accuse others of lying if they have figures that don’t agree with the ones they prefer? How about a little humility, Willis? Do you think it could be possible that *your* preferred figures could be wrong – or have you ruled that out?

      http://earthobservatory.nasa.gov/Features/CarbonCycle/page5.php
      “Carbon dioxide causes about 20 percent of Earth’s greenhouse effect; water vapor accounts for about 50 percent; and clouds account for 25 percent. The rest is caused by small particles (aerosols) and minor greenhouse gases like methane.”

      “Water vapour is the primary greenhouse gas in the Earth’s atmosphere. The contribution of water vapour to the natural greenhouse effect relative to that of carbon dioxide (CO2) depends on the accounting method, but can be considered to be approximately two to three times greater.” – IPCC AR5 Chapter 8, page 666

      http://eae.sagepub.com/content/25/8/1439.abstract
      Each method shows that, on average, water vapour contributes approximately 96% of current greenhouse gas warming. Thus, the factors controlling the amount of water vapour in the air also control the earth’s temperature.

      TOTAL BACK RADIATION OF ALL GHG Figure 7 is FAQ 1.1 Figure 1 from page 96 of AR4. It shows the radiation balance for the earth and that the back radiation of all of the greenhouse gases is 324 W m-2. This is the value used to calculate the RF [radiative forcing] of CO2 at 378 ppmv as (8.67/324)/100 = 2.7% back radiation of the total of all of the greenhouse gases

      From Table 1, CO2 accounts for 2.7% of the global warming while all of the other gases account for approximately 0.7% for a total of approximately 3.4%. It becomes evident that, on average, water vapour accounts for approximately 96% of the current global [greenhouse effect] warming. This is an important finding because it leads to the conclusion that the factors controlling the average level of water vapour in the atmosphere also control atmospheric temperature.
      [O]n average, each molecule of CO2 is surrounded by approximately 23 molecules of water vapour at ground level. … If the warming effect of water molecules and CO2 molecules were the same, then the contribution of CO2 would be (1/22.7) = 4.4% of that of water vapour. But from the previous section, water molecules are 1.6 times more effective at warming than CO2 molecules. Using this value and the ratio of 22.7:1, the contribution of CO2 to warming of the atmosphere is approximately (1/22.7)/1.6 = 2.8% of that of water vapour. As water vapour is approximately 96% of the total RF of all of the GHG, the contribution of CO2 is approximately 4% less than this, i.e., 2.69%. If the average RH were 60%, the contribution of CO2 would be ((1/27.4)/1.32) x 0.96 = 2.65%. For practical purposes, these values are the same as the 2.7% obtained by the quadratic model.

      1. BobW in NC

        Kenneth – Thank you so much for this information. It is exactly the level and detail I have been looking for. It is the most definitive I have seen.
        BobW in NC, a biologist

    2. cementafriend

      Clearly Willis does not understand emission and absorption of radiation by gases. Modtran is a computer model that has some false assumptions -there have been a number of peer reviewed papers that have critised it.
      Prof Hoyt Hottel Prof of Chemical Engineering at MIT) made a huge amount of measurements of radiation emission and absorption by flames and in heat exchangers to determine data for the design of furnaces. Some of the important data can be found in Chapter 5 (Heat & Mass Transfer) of Perry’s Chemical Engineering Handbook. There is an equation y which you can calculate the absorptivity of gas mixtures. The Stefan-Boltzmann equation applies to solids and not gases but Hottel found that one could relate the equation on a volume basis if one considered the partial pressure of the gas and the path length of the radiation. For Willis’s benefit at the same temperature of 100F and the same partial pressure-path length product the emission of water vapor is about 4 times that of carbon dioxide gas. Absorption depends on the temperature and emissivity of the source. The absorptivity will be in the same ratio if a black body at 100F was radiating to water vapor or CO2 at lower temperature than the source.
      However in the atmosphere water vapor makes up 2 to 4% while CO2 is only now about 400ppm (or 0.004%) Without considering any overlap. The actual absorption of radiation from the earth surface by CO2 is so small to be not measurable particularly as the average surface temperature is around 42F. Of the radiation emitted from the surface and absorbed by the atmosphere it is correct to say over 98% will be absorbed by water vapor.

  4. rferris

    Willis need to up his game on sources of information. He also might want to study a little physics so he can tell malarkey from science….he clearly will believe any nonsense he reads at his preferred sites.
    It takes very little education to laugh at the numbers you present!!!
    At .002% of the atmosphere CO2 absorbs all the radiation that falls in its absorption bandwidth. Nothing special after that.
    To believe a gas that is only .04% of the atmosphere could absorb 41% of the radiation that fall on the planet is scientific illiteracy.
    Willis has proven he WILL believe things that are made up if they agree with his preconceived notions.

    1. Kenneth Richard

      “Willis has proven he WILL believe things that are made up if they agree with his preconceived notions.”

      Confirmation bias is “the tendency to search for, interpret, favor, and recall information in a way that confirms one’s preexisting beliefs or hypotheses, while giving disproportionately less consideration to alternative possibilities.” — Wikipedia

      Not only does Willis not consider alternative possibilities to his perceptions of “truth,” he claims that those who disagree with his views are just “making stuff up”, or lying.

      Ramanathan et al., 1989 (1,200+ citations)
      http://www.indiaenvironmentportal.org.in/files/file/cloud%20radiative%20forcing.pdf
      “Water vapour and cloud are the dominant regulators of the radiative heating of the planet. ..The greenhouse effect of clouds may be larger than that resulting from a hundredfold increase in the CO2 concentration of the atmosphere.”

      “The size of the observed net cloud forcing is about four times as large as the expected value of radiative forcing from a doubling of CO2. The shortwave and longwave components of cloud forcing are about ten times as large as those for a CO2 doubling.”

      http://multi-science.atypon.com/doi/abs/10.1260/0958-305X.25.2.389
      “We will show that changes of relative humidity or low cloud cover explain the major changes in the global mean temperature. We will present the evidence of this argument using the observed relative humidity between years 1970 and 2011 and the observed low cloud cover between years 1983 and 2008. One percent increase in relative humidity or in low cloud cover decreases the temperature by 0.15 °C and 0.11 °C, respectively. In the time periods mentioned before the contribution of the CO2 increase was less than 10% to the total temperature change.”

  5. Frederick Colbourne

    I have Dr. Salby’s textbook and have watched two of his lectures, including the technical lecture he gave to other scientists.

    In my opinion, Dr. Salby has merely outlined his case, but he has not made public a full exposition.

    Dr. Salby has shown where the potential uncertainties in CO2 sources and sinks may lie and he has demonstrated that phase relationships are relevant to estimating imbalance between sources and sinks, probably at all time scales.

    But unless Dr. Salby receives support from a public or private institution, he may be unable to develop his ideas.

    In the meantime, while we hope for further exposition Dr. Salby, there are other active lines of research.

    Climate sensitivity is one promising avenue.

    J. Ray Bates, Estimating Climate Sensitivity Using Two-zone Energy Balance Models, 2015.
    http://onlinelibrary.wiley.com/doi/10.1002/2015EA000154/pdf

    Cloud and water vapor feedbacks are closely related to climate sensitivity via variability in Earth’s albedo.

    The very fact that the leading climate models (GCMs) have not converged in 35 years indicates there is complete lack of consensus in the treatment of both water vapor and clouds.

    J. Ray Bates discusses this subject in the paper cited.

    In my opinion, clouds and their role in albedo hold the key to climate change, more so than CO2.

  6. Val

    “They cannot both be correct.”

    It follows that

    a. proxy temperature is correct but proxy CO2 is incorrect or

    b. proxy temperature is incorrect bur proxy CO2 is correct or

    c. they’re both incorrect.

    Place your bets now.

    1. Kenneth Richard

      Considering we have substantial records of absent or severely recessed glaciers from warm periods, farming and colonization on Greenland during a 350-year period (~900-1250 AD), grapes grown in Scotland, trees in the Arctic…ice fairs on the River Thames, substantial records of crop failures and shorter growing seasons during the Little Ice Age…. there is a greater likelihood that temperature proxies that match up well with these warm/cool periods are more accurate than CO2 concentration values. Seems like every year we get different CO2 concentration values for past climates. Just a few million years ago, CO2 levels reached 470 ppm as of 2016 published research. Past research said CO2 hasn’t reached 400 ppm for 10s of millions of years.

      http://www.sciencedirect.com/science/article/pii/S0012821X16000388
      During the past five million yrs, benthic δ18O records indicate a large range of climates, from warmer than today during the Pliocene Warm Period to considerably colder during glacials. Antarctic ice cores have revealed Pleistocene glacial–interglacial CO2 variability of 60–100 ppm, while sea level fluctuations of typically 125 m are documented by proxy data. … Our model shows CO2concentrations of 300 to 470 ppm during the Early Pliocene [~5 million years ago]. Furthermore, we simulate strong CO2 variability during the Pliocene and Early Pleistocene. These features are broadly supported by existing and new δ11B-based proxy CO2 data, but less by alkenone-based records. The simulated concentrations and variations therein are larger than expected from global mean temperature changes. Our findings thus suggest a smaller Earth System Sensitivity than previously thought. This is explained by a more restricted role of land ice variability in the Pliocene. The largest uncertainty in our simulation arises from the mass balance formulation of East Antarctica, which governs the variability in sea level, but only modestly affects the modeled CO2 concentrations.

  7. TedM

    “Is that really how scientists are supposed to conduct themselves – accuse others of lying if they have figures that don’t agree with the ones they prefer? How about a little humility, Willis?

    To your first sentence Kenneth Willis isn’t a scientist, to the second sentence I doubt that humility is in his vocabulary.

    Your post was an excellent rebuttal. Hoping that Willis discovers the humility to read it.

    1. DirkH

      Also, I started Modtran and I didn’t find the values Willis mentioned in its output, with the default settings. Willis, how do you arrive at your numbers?

      1. stan stendera

        I’m suspicious, from having read much of Willis on WUWT, the this just might be a Willis impostor. He does not frequently comment on non WUWT blogs.

        1. yonason
    2. yonason

      “…Willis isn’t a scientist,…” – TedM

      Basically, yeah.
      http://www.populartechnology.net/2013/10/who-is-willis-eschenbach.html

      1. Colorado Wellington

        “…Willis isn’t a scientist,…”

        I wouldn’t go there, yonason. A scientist is anyone using the scientific method to explore nature, not credentialing. Let’s not sanction the idea popular among the nitwits who insist that only credentialed “scientists” have the standing to participate.

        I don’t like Eschenbach’s sloppy personal attack on Salby (if it is in fact him) and it reflects poorly on him.

        “Professor Salby is simply making things up and hoping that you’ll believe them.”

        I’ve read some abrasive things from Willis before but I don’t recall him speculating about another man’s state of mind. This sounds really idiotic. I’ll be disappointed if the real Willis Eschenbach wrote the above sentence.

        1. yonason

          “I wouldn’t go there, yonason.”

          Gotta disagree with you here, C.W. Graduate school is, like med school, advanced training to assure that one has the comprehensive tools to practice one’s discipline. Sure, it’s not a guarantee you won’t have a percentage who are guilty of malpractice, but it is nevertheless necessary to ensure an appropriate level of quality in most cases.

          E.g., Willis doesn’t have even the remotest grasp of testing hypotheses, which is a fundamental principle. If he thought it, it’s true, even though someone else has tested it and found it didn’t work. A good scientist is always skeptical, even of his own ideas. That’s not Willis, by any stretch.

          I do see your point, and wouldn’t care if Willis was respectful of those who would probably be willing to help him improve (he is a bright guy), if his ego didn’t always get in the way. But he hasn’t even the most rudimentary concept of what science is, and that makes him a loose cannon.

          Whether that’s his comment or not won’t change what I’ve written, either. And if that’s someone imitating him, they do a bang on representation.

        2. yonason

          Years ago I took some engineering courses (heat and mass transport among them), and the prof told us about a housewife who learned the math, and used to write articles about the physics of soap bubbles she observed while doing the dishes. She was so good, she became a recognized authority, and was given some award. Can’t remember who, or find a ref about, but it’s proof that among the qualifications to be recognized as a scientist, one mus produce something useful to others. Oh, and I doubt she went around telling the gentlemen of the day that they were full of it because they didn’t agree with her.

          Wake me when Eschenbach makes a notworthy contribution.

          1. Colorado Wellington

            A “notworthy” contribution is a fantastic Freudian slip, yonason. 🙂

            —–
            P.S. We agree on much.

            I’m an engineer and I know that professional rigor is based on proper training and practice. I just despise smug alarmists claiming supremacy because their critics are not the right kind of “scientists” and “specialists”.

            Jeff Masters is a case in point. Even in his “towering figure” eulogy to Bill Gray he couldn’t refrain from calling CAGW skeptics “climate deniers” and making an argument that he was “not a climate change specialist”.

  8. Ron C.

    Kenneth, thanks a lot for the link to Salby’s testbook: much to read and digest. And thanks also for your comments here, also with sources and links.

    1. Kenneth Richard

      Quite welcome, Ron. I want to thank *you* for the discovery-post (NSIDC) from back in December or so that essentially said internal oscillatory changes in the ocean and sea ice precede changes in the atmosphere, not the other way around, and that variations in sea ice are thus a reflection of the internal natural variability of the polar climate system, not a reflection of atmospheric forcing. Very enlightening.
      —–
      https://nsidc.org/pubs/special/16/NSIDC-special-report-16.pdf
      Thus, the presented facts suggest that the most significant cause of changes in the ice cover extent are the changes in the vertical water structure in the upper ocean layer, rather than the changes of thermal conditions in the atmosphere. These changes are induced by fluctuations in the horizontal dimensions of the halocline, which are governed in turn by the expansion or reduction of the surface Arctic water mass. Pg. 49

      Conclusion:
      However, the order of the climate signal transfer in the climate system, when changes in the ocean induced by the disturbances of its freshwater balance precede changes in sea ice extent, and the latter precede changes in the atmosphere, suggests that self oscillation is the most probable cause for the development of natural processes at the present time. The idea, that the internal dynamics of the climate system are the main moving force in the development of the natural process, was used to account for vast glacial-interglacial fluctuations in the Pleistocene.

      The remarkable stability of our planetary climate system derives from feedbacks between internal parts of the system, providing the oscillations we observe as natural variability. Arctic Sea Ice is a prime example.

  9. Martin Hertzberg

    “Back radiation” from the colder atmosphere to the warmer Earth below is impossible: itviolates the 2nd Law. Too bad that an excellent analysis of atmospheric sources and sinks of CO2 is marred by any consideration of the undefined, fiction called the “greenhouse effect.” For agreement with Salby’s analysis for the relative insignificance of atmospheric CO2 and the human emission of CO2 in
    determining the climate, see Hertzberg and Schreuder “Reassessing the
    Climate Role of Carbon Dioxide”. For a simple analysis of the effect of clouds on temperature, go to Hertzberg “Earth’s Radiative Equilibrium in the Solar Irradiance”, E&E Vol 21,No. 9 2009 pp 83-93. See “Slaying the Sky Dragon – Death of the Greenhouse Gas Theory”, Stairway Press, 2011

    1. yonason

      “‘Back radiation’ from the colder atmosphere to the warmer Earth below is impossible: itviolates the 2nd Law.” – Martin Hertzberg

      Technically that statement is incorrect. There is always an exchange of heat photons between objects of different temperature. What is important is that the NET flux of photons is ALWAYS from the warmer to the colder.

      In the absence of a heat source, the “back radiation” merely slows loss of heat from the earth to space. When the earth emits heat, it cools, and when a portion of the heat it emitted is returned, it rewarms, but never gets as warm as it was before, because not all the heat is returned.

      I take it that’s what you meant, but I think in this case it’s important to be specific.

      “the devil is in the details” after all.

  10. Peter

    “Warming of SST (by any mechanism) will increase the outgassing of CO2 while reducing its absorption. Owing to the magnitude of transfers with the ocean, even a minor increase of SST can lead to increased emission of CO2 that rivals other sources”

    As a layman, I’ve never been able to understand how the oceans are supposed to be absorbing more co2 (becoming more “acidic”) if they are warming. Warming is supposed to result in a net out gassing of co2 causing co2 to rise.

    is co2 outgassing effected by atmospheric co2 concentration?

    Can someone please explain?

    1. Ferdinand Engelbeen

      Peter,

      In absence of an external source, there is a dynamic equilibrium between CO2 in the oceans and in the atmosphere (over long term vegetation plays a minor role). That is governed by Henry’s law, which says that at a given temperature, there is a fixed ratio between the partial pressure of a gas above a solution and what is dissolved in the solution. That is as good the case for a closed sample in a lab as for the oceans.

      In the latter case, it is more complicated as the exchange is not “static” as in the lab sample, but “dynamic”: lots of CO2 are emitted at upwelling places where the cold, CO2 rich waters from the deep oceans are warming up near the equator and lots of CO2 are sinking near the poles together with the sinking waters there (both around 40 GtC/year) into the deep oceans. As long as these in and out fluxes are equal, the air-ocean system is in “steady state” or dynamic equilibrium.

      Now the natural steady state between atmosphere and ocean surface for the current average seawater surface temperature (~15°C) would be around 290 ppmv. The current CO2 pressure in the atmosphere is 110 ppmv above steady state, thus the net CO2 flux is from the atmosphere into the oceans, not reverse… That affects both the upwelling (less) and the sinking (more) of CO2. The net effect is about 3 GtC/year more sink than source over a permanent circulation of ~40 GtC/year between atmosphere and deep oceans.
      That also affects total CO2 in the ocean surface (increasing) and pH (decreasing), which were measured at a few places (Hawaii, Bermuda) over longer periods.

      From the ~9 GtC/year CO2 that humans emit as mass, some 1 GtC/year is absorbed by the biosphere (more growth of plants than decay, the earth is greening), ~0.5 GtC/year by the ocean surface (which has a limited capacity) and ~3 GtC/year by the deep oceans, which have an enormous capacity, but a limited direct exchange with the atmosphere.

      1. AndyG55

        You are not a luke-warmer..

        you are rabid alarmist.

        1. Ferdinand Engelbeen

          AndyG55, a realist is someone who accepts what is measured and looks at the possible consequences:

          A change of 0.1 pH unit over 160 years is of no consequence as the pH over the oceans differs several units in river estuaries, depending of what water passes by, about one pH unit from poles to equator and even within a coral reef can change 1 unit over a day.

          For an alarmist, 0.1 pH unit change over 160 years caused by humans is a catastrophe, no matter the (non) consequences.

          For a d*n**r, no human emissions can be even measurable or can have consequences…

          Take your pick…

          1. Kenneth Richard

            “For a d*n**r, no human emissions can be even measurable or can have consequences…”

            Looks like you’ve just concocted a straw man here.

          2. Ferdinand Engelbeen

            Kenneth,

            It is with much regret that I am too often confronted with statements from people who call themselves “skeptics” which are simply wrong, proven wrong many times and still repeated at nausum in the blogosphere:

            – Human CO2 is only 3% of all emissions, thus only 3% of the increase in the atmosphere is from humans.
            – CO2 is only 0.04% of the atmosphere, thus too small to have any effect.
            – Greenhouse gases don’t exist.
            – Greenhouse gases have no warming effect on temperature.
            – CO2 always follows temperature, never leads, thus can’t be the cause of the recent temperature increase.
            – The warming oceans are the only source of the extra CO2 in the atmosphere.
            – The extra CO2 is from the warm Medieval period and now is coming back with an 800 year delay (as seen in the ice cores).
            – Mauna Loa is on a volcano, thus…

            and many others…

            All quite wrong arguments. Used in any debate you loose the debate even before started as that simply proves that you don’t know where you are talking about.

            The real debate is about the real effect of CO2: the climate sensitivity for a CO2 doubling, which is much lower than what all models “project”. No disaster, only benign for plants, animals and humans…

          3. sod

            “– Human CO2 is only 3% of all emissions, thus only 3% of the increase in the atmosphere is from humans.
            – CO2 is only 0.04% of the atmosphere, thus too small to have any effect.
            – Greenhouse gases don’t exist.
            – Greenhouse gases have no warming effect on temperature.
            – CO2 always follows temperature, never leads, thus can’t be the cause of the recent temperature increase.
            – The warming oceans are the only source of the extra CO2 in the atmosphere.
            – The extra CO2 is from the warm Medieval period and now is coming back with an 800 year delay (as seen in the ice cores).
            – Mauna Loa is on a volcano, thus…”

            all of these opinions are supported by people posting here. Sometimes i am happy that i am a warmist, as i doubt that i could handle this much confusion on my side…

          4. Kenneth Richard

            I agree that those statements – especially the way they’re worded – are not consistent with conventional scientific conclusions. This is especially true when you intentionally insert words like “only” and “never” in there so as to make the statement as extreme and outrageous as possible. In the future, it would be best to leave out those quantitative extremes when attempting to concoct your assumptions about what is said or has been said by skeptics of the “consensus” opinions.

            I agree that CO2 is a greenhouse gas and plays a (minor) role within the greenhouse effect, a role that logarithmically decreases as the concentration increases. I agree that CO2 both follows and leads temperature, though it mostly follows per observation. I agree that humans have contributed to the overall increase in atmospheric concentration, though in proportions much smaller than 100%, and probably well below 50% too. (Even Salby acknowledges the contribution from humans could possibly be as high as 30%.) But this is all highly speculative, and far from “settled” science. I’m OK operating within that uncertainty.

            I agree that climate sensitivity is what it all comes down to. Doubling CO2 to 560 ppm is theoretically supposed to result in 1.2C of warming, with positive feedbacks with cloud/water vapor “believed” (the word the IPCC actually uses) to amp that 1.2C up to 3.0C and beyond. The feedbacks, however, turn out to be net negative, especially with cloud changes – with clouds far more dominant within the GHE than CO2. The net negative feedbacks may result in a negligible overall change (<1C)due to CO2 increases. The IPCC "believes" feedbacks are net positive due to modeling and presupposition, and therefore they can claim ECS is much scarier than the change theoretically attributed to CO2 alone (1.2C). I've personally compiled a list of nearly 50 peer-reviewed papers that support low climate sensitivity. Yet another one (ECS = 1C or less) was just published within the last few days.

            I can understand why you'd want to "correct" those who express an only-never view of anthropogenic or CO2 impacts on climate. I'm not one who expresses those views that humans have *no* effect on climate, or that the CO2 changes are *all* natural. You should probably avoid making those assumptions until they have been confirmed by reading what someone has actually written.

      2. Bart

        The fact that there is a dynamic flow of CO2 within the THC produces a temperature dependence on the rate of change of CO2 concentration in the upper oceans, and thence to the atmosphere. It is almost perfectly modeled in the modern era by the differential equation

        dCO2/dt = k*(T – T0)

        k = coupling factor in ppmv/K/unit-of-time
        T = temperature anomaly
        T0 = affine offset parameter

        The relationship is still holding well with the latest El Nino:

        http://woodfortrees.org/plot/esrl-co2/derivative/mean:12/from:1979/plot/rss/offset:0.6/scale:0.22

        It will also hold in the coming La Nina plunge. Watch for it.

        1. Ferdinand Engelbeen

          Hi Bart,

          Some time ago…

          Still fixed at your one equation fits all I see, which is inconsistent with every observation on earth, including Henry’s law which says that dCO2/dt is directly in ratio to the pCO2 difference between atmosphere and ocean surface.

          That means that for a fixed temperature step, the increase in pCO2(water) of ~16 μatm/°C is fully compensated with a 16 ppmv/°C increase in the atmosphere, restoring the in/out fluxes at every point of the oceans to what they were before the increase in temperature…

          1. Bart

            “That means that for a fixed temperature step, the increase in pCO2(water) of ~16 μatm/°C is fully compensated with a 16 ppmv/°C increase in the atmosphere, restoring the in/out fluxes at every point of the oceans to what they were before the increase in temperature…”

            This is not a static problem. The ocean is not a stagnant pool of water.

          2. Ferdinand Engelbeen

            Bart,

            The CO2 in/out fluxes at EVERY point of the ocean surface are restored with only a 16 ppmv increase in the atmosphere for 1°C increase everywhere of the oceans surface. That is as good for the huge CO2 input fluxes at the upwelling places near the equator as for the huge output fluxes into the sinking waters near the poles and everywhere else where ocean surface and atmosphere meet.
            I call that a response of a dynamic process to changes in temperature and pressure. You may have a different definition of “dynamic”, but mine is used by plenty of scientists…

          3. Bart

            “The CO2 in/out fluxes at EVERY point of the ocean surface are restored with only a 16 ppmv increase in the atmosphere for 1°C increase everywhere of the oceans surface.”

            No, that is only the static effect. Beyond that, upwelling CO2 laden waters are bringing new parcels in every second, while downwelling waters are carrying less down. CO2 accumulates in the surface oceans, and thence to the atmosphere.

          4. Bart

            Don’t know what happened to my reply. If it spontaneously appears later, will give more detail, but you are doing static analysis.

          5. Ferdinand Engelbeen

            Bart,

            What is the difference between “static” and “dynamic”? None in this case.

            What is upwelling is rich in CO2 and when warming up it will release CO2, each new parcel an amount in ratio to the pCO2 difference with the atmosphere. The opposite happens at the sink places.

            If the temperature at the upwelling places increases with 1°C, the amount released will increase (less than 5%) and for the same temperature increase at the sink places the amount taken away will decrease (with less that 5%). Both give an increase of CO2 in the atmosphere. So far we agree.

            When the CO2 pressure in the atmosphere increases, the opposite happens: less is released at the upwelling places and more taken away at the sink places. That is completely ignored in your formula. When reaching 16 ppmv/°C extra in the atmosphere, the original in/out fluxes are restored and we are back at the situation as before the temperature increase, be it at a higher CO2 level.

            That is exactly the same increase as for a closed sample in a lab per Henry’s law: the same CO2 change for the same temperature change for single static sample as for the whole dynamic oceans…

            See: http://www.ferdinand-engelbeen.be/klimaat/klim_img/upwelling_temp.jpg

          6. Bart

            No, Ferdinand.

            This is not a static pond of water. Henry’s law does not apply directly. You’ve got to take account of the fact that external CO2 is being introduced continuously into the system. If it is not being transported back down with downwelling waters at the same rate it is being brought up in the upwelling waters, it will accumulate. There is no alternative.

        2. Ferdinand Engelbeen

          Forgot to add the graph based on the linear response of CO2 sinks to the increased CO2 pressure in the atmosphere and the response of mainly vegetation to temperature variability:

          http://www.ferdinand-engelbeen.be/klimaat/klim_img/had_co2_emiss_nat_deriv.jpg

          The response of the sinks to the increased pressure in the atmosphere still is in the middle of the variability, except for huge extremes:

          http://www.ferdinand-engelbeen.be/klimaat/klim_img/dco2_em6.jpg

          La Niña will align that again in the middle…

          1. Bart

            “La Niña will align that again in the middle…”

            We shall see…

    2. Kenneth Richard

      Yes, you’ve hit on a major contradiction in the “ocean acidification” paradigm.

      http://www.principia-scientific.org/ocean-acidification-claims-are-misleading-and-deliberately-so.html
      “Either the oceans are getting warmer and the CO2 concentration in seawater is decreasing, which means that ocean acidification from man-made CO2 from the atmosphere is nonsense. Or the oceans are getting cooler and the man-made CO2 from the atmosphere is dissolving in those cooler oceans and causing – insignificant – ocean acidification, which means that warming oceans and the associated sea level rises are nonsense.”

      The oceanic biosphere is a net source of CO2. Corals, for example, when healthy, release more CO2, thus “acidifying” their own habitat.

      https://www.newscientist.com/article/dn28468-growing-corals-bathe-themselves-in-acid-without-suffering-damage/?utm_source=NSNS&utm_medium=SOC&utm_campaign=hoot
      More acidic water may be a sign of healthy corals, says a new study, muddying the waters still further on our understanding of how coral reefs might react to climate change. Andreas Andersson of the Scripps Institution of Oceanography in San Diego, California, and his colleagues carefully monitored a coral reef in Bermuda for five years, and found that spikes in acidity were linked to increased reef growth. … The team found that coral growth itself made the water more acidic as the corals sucked alkaline carbonate out of the water to build their skeletons. The corals also ate more food during these high-activity periods and pumped more CO2 into the water, increasing acidity further.”

      http://onlinelibrary.wiley.com/doi/10.1002/2015GL064431/abstract
      “We find that ocean pH does not simply reflect atmospheric CO2 trends, but rather that circulation/biogeochemical changes account for >90% of pH variability in the Sargasso Sea and more variability in the last century than would be predicted from anthropogenic uptake of CO2 alone.”

      Look closely at this IPCC graph (below link). Notice the far left black incoming and outgoing arrows, which show that the oceans themselves emit or release 78.4 gigatons of carbon (GtC) per year, and they absorb 80 gigatons of carbon per year. (These are little more than wild guesses, by the way, pre-designed to fit the humans-cause-CO2-concentration-increases scenario.)

      http://www.climatechange2013.org/images/figures/WGI_AR5_Fig6-1.jpg

      Now look at the red arrows in the middle of the graph, which show that human emissions contribute 8.9 gigatons of carbon to the atmosphere (7.8 fossil fuels, 1.1 land use changes) per year. Of this 8.9 GtC per year emitted by humans (which is ~1/9th of that emitted by the oceans themselves), the IPCC says that 26% of it is absorbed by the oceans (the rest by land sinks and the atmosphere). That’s about 2.3 gigatons of carbon absorbed by the oceans due to anthropogenic emissions. So, of the 80 gigatons of carbon absorbed by the oceans, just 2.3 GtC of it comes from human sources. So why do we presume that the oceans are “acidifying” (or experiencing pH flux anyway) due to human emissions when the oceans themselves release (and absorb) 30 to 40 times as much GtC as humans contribute?

      1. Ferdinand Engelbeen

        Kenneth Richard,

        The effect of temperature at equilibrium is less remaining CO2 and higher pH of the surface waters for higher temperatures, but the ocean-atmosphere system is not in equilibrium yet: there is far more CO2 in the atmosphere, 100 ppmv above steady state, for the current (area weighted) average ocean surface temperature. That makes that the net CO2 flux is from the atmosphere into the oceans, despite the increased surface temperature.

        That was measured at some 6 stations where regular measurements were taken, of which the longest series are at Bermuda and Hawaii.

        Here the graphs for Hawaii:
        http://www.pnas.org/content/106/30/12235.full.pdf
        and here for Bermuda:
        http://www.biogeosciences.net/9/2509/2012/bg-9-2509-2012.pdf
        where DIC represents the sum of all inorganic carbon species (free CO2 + bicarbonates + carbonates) which are easily converted to CO2 and back, depending of the pH.

        Feely e.a. have compiled over 2 million ocean surface measurements into a global uptake of CO2 by the oceans. See:
        http://www.pmel.noaa.gov/pubs/outstand/feel2331/exchange.shtml
        and following sections.

        1. Kenneth Richard

          Ferdinand: “there is far more CO2 in the atmosphere, 100 ppmv above steady state”

          There is 50 times more CO2 stored in the oceans than in the atmosphere. Even a tiny adjustment in temperature is enough to perturb the oceanic outgassing or retention “balance” at a rate that far exceeds the 1/100ths of 1% change (0.01%) in the the atmospheric CO2 concentration (+0.0001, or +1 part in 10,000) since 1900, of which only a tiny fraction (2-3%) is even claimed to be possibly due to anthropogenic sources according to the IPCC (see the above source).

          Ferdinand, can you explain why you believe pH variability can and should predominantly be attributed to human fossil fuel consumption when scientists (Goodkin et al., 2015) have determined that “circulation/biogeochemical changes account for >90% of pH variability in the Sargasso Sea”?

          Also, considering that pH has reportedly decreased by a net 0.07 to 0.08 in the last 200 years (Wei et al., 2015), but that, in a span of just a few years, it may fluctuate by 2 to 5 times that long-term amount, can you explain wherein lies the danger in pH variability in those statistically insignificant ranges?

          https://www.researchgate.net/profile/Gangjian_Wei/publication/283946141_Decadal_variability_in_seawater_pH_in_the_West_Pacific_Evidence_from_coral_d_11_B_records/links/5653244708aefe619b191e48.pdf
          “We here estimate the OA rates from the two long (>150 years) annually resolved pH records from the northern SCS (this study) and the northern GBR [Wei et al., 2009], and the results indicate annual rates of -0.00039 +/- 0.00025 yr and -0.00034 +/- 0.00022 yr for the northern SCS [South China Sea] and the northern GBR [Great Barrier Reef], respectively. It is worth noting that the errors of these estimates are fairly large with RSD of 65% for that these two time-series do not show significant decreasing trend for pH. Despite of such large errors, estimated from these rates, the seawater pH has decreased by about 0.07–0.08 U over the past 200 years in these regions.”

          [Fig. 2, page 5, red font shows pH varying more in a span of a few years – 0.1 to 0.4 – than it has for the entirety of the last 200 years (0.07 to 0.08).]

          1. Ferdinand Engelbeen

            Kenneth Richard,

            How much CO2/derivatives are in the total oceans is of zero interest for the equilibrium with the atmosphere. Only the temperature and concentrations at the surface are of interest:
            It doesn’t matter if you shake a 0.5, 1.0 to 1.5 liter bottle of Coke from the same batch: the pressure under the screw cap will be the same at the same temperature (except for a relative larger loss out of the liquid for the smaller bottle)…

            The deep oceans are largely isolated from the surface and play a minor role on -relative- short periods, they are dominant over long periods like the glacial – interglacial transitions (with atmospheric increase “speeds” like 0.02 ppmv/year)…

            Common error of many skeptics is comparing the -indeed- tiny contribution of humans to total natural emissions, but at the same time forgetting to account for the larger natural sinks… It is like a bookkeeper only accounting for the gains and not counting the larger losses of the factory. No shareholder would be happy with that.

            Human emissions are one-way surplus, while the largest part of the natural cycle is seasonal, where hardly any change in net balance can be found over the past 55 years of accurate measurements: always more sink than source, thus not the cause of the increase…

            “circulation/biogeochemical changes account for >90% of pH variability in the Sargasso Sea”

            The problem is in in the word “variability”: human emissions show hardly any variability, only a steady, slightly quadratic increase over time. So do levels in the atmosphere and so do levels in the ocean surface (and so do inversely δ13C levels in atmosphere and ocean surface). See the change in δ13C levels of coralline sponges at Bermuda over the past 600 years, compared to atmosphere:
            http://www.ferdinand-engelbeen.be/klimaat/klim_img/sponges.jpg

            Thus indeed most seasonal to decadal variability in pH is non-human, but the trend is human.
            That is the same problem as for tide gauges: one need at least 25 years of measurements to -statistically- separate the trend in sea level change from the much larger changes caused by waves and tides, but that doesn’t disprove that there is a trend…

          2. AndyG55

            ““there is far more CO2 in the atmosphere, 100 ppmv above steady state”

            Your so-called steady state of around 280ppm is the point below which most plants stop functioning.

            It is a disastrously LOW amount of CO2 for the atmosphere.. the world on the very brink of survival

            Only a monumental fool would think this is a “steady state”.

          3. Ferdinand Engelbeen

            AndyG55,

            Steady state has nothing to do with what is good or bad for plants. Steady state only says that for the current geological time frame of a few million years at the current average temperature of the ocean surface, the amount of CO2 entering the atmosphere and leaving the atmosphere was the same if the atmosphere is at 290 ppmv (280 ppmv pre-industrial).

            The lowest -dangerous- level where C3 plants can survive is 180 ppmv. That was encountered over long periods during glacials. Fortunately CO2 levels near ground over land are 30-40 ppmv higher than in the bulk of the atmosphere, due to vegetation decay, so that at least during a few hours in the morning photosynthesis was possible.

            Thus 280 ppmv was already better and 1000 ppmv is regularly used in greenhouses as most plants grow faster at that level…

          4. Kenneth Richard

            Ferdinand: “Common error of many skeptics is comparing the -indeed- tiny contribution of humans to total natural emissions, but at the same time forgetting to account for the larger natural sinks”

            The problem you’re having, Ferdinand, is that you’re wholly stuck in the non-skeptical arena of presuming that the guesses and beliefs about natural sinks relative to sources of CO2 are both known and certain enough to leap to the conclusion that the natural sinks for CO2 are larger than – or in nice, neat equilibrium with – the natural sources, conveniently allowing for the paradigm that says anthropogenic CO2 emissions, tiny as they are, to be 100% (or close to it) cause of atmospheric CO2 increases.

            Can you explain why it’s your belief that we’ve got the figures for natural sources and sinks all figured out, allowing you to rest assured that the IPCC’s stated natural sink/source figures are both reliable and correct? We routinely “discover” new sources and sinks of CO2 that we had never knew existed before. So why do you think our current figures for sources and sinks are nonetheless reliable and correct – since they necessarily must change every time we “discover” a new source or sink that had not been known to exist before?

            Summarizing, can you understand how *un*-skeptical your quoted statement above is? Skeptics question. You’ve wholly accepted the beliefs stated by the IPCC about sources and sinks, and have decided those beliefs are true. Yours is not a skeptical position.

            Ferdinand: “Thus indeed most seasonal to decadal variability in pH is non-human, but the trend is human.”

            The pH “trend” amounts to -0.07 to -0.08 over the last 200 years, with uncertainties/errors about as large as that presumed “trend.” Do you consider this “trend” to be statistically significant?

            Wei et al., 2015: “[R]esults indicate annual rates of -0.00039 +/- 0.00025 yr and -0.00034 +/- 0.00022 yr for the northern SCS [South China Sea] and the northern GBR [Great Barrier Reef], respectively. It is worth noting that the errors of these estimates are fairly large with RSD of 65% for that these two time-series do not show significant decreasing trend for pH. Despite of such large errors, estimated from these rates, the seawater pH has decreased by about 0.07–0.08 U over the past 200 years in these regions.”

            Furthermore, do you have hard evidence you can point to that says the pH “trend” that more or may not exist due to the high degree of uncertainty/error is most assuredly caused by humans? Because that sounds like only a belief of yours. I’m a skeptic. I need more than presumptions that fall in line with a “consensus.”

            And can you answer my question about what the dangers are in a -0.07 to -0.08 pH “trend” over the course of 200 years when in the span of a few years pH “naturally” changes by +/- 0.1 to 0.4, which is about 2 to 5 times the total net change, or “trend”, for the last 200 years that you believe is caused by humans? Whence comes the harm?

          5. Ferdinand Engelbeen

            Kenneth,

            The difference between you and me is that I am (near) as critical to what is said by skeptics like Dr. Salby and others as to what is said by the warmistas (IPCC, Mann, Trenberth, Karl,…).

            Most people, myself included, have a tendency to uncritically accept what is said by others if that fits their own ideas, but I remain skeptic whatever the source…

            About the carbon cycle…
            There is no need at all to have any knowledge of any individual natural in or out flux, because we have quite accurate knowledge of the sum of all natural CO2 fluxes together: that is reflected in the variability and height of the increase in the atmosphere:

            increase = human emissions + natural sources – natural sinks

            We have reasonable knowledge of human emissions, based on fossil fuel sales (taxes!), +/- 1 GtC. We have accurate measurements (+/- 0.4 GtC) of the CO2 increase in the atmosphere. For the past years that gives per year:

            4.5 GtC = 9 GtC + X – Y

            X – Y = -4.5 GtC/year or about 2.15 ppmv/year more sink than source.

            It doesn’t matter at all that in any year:
            X = 10 GtC and Y = 14.5 GtC or
            X = 100 GtC and Y = 104.5 GtC or
            X = 1000 GtC and Y = 1004.5 GtC
            or that everything was absorbed in the oceans in one year and vegetation was a net source in the same year and the next year it is reverse… X and Y are totally unimportant, only X-Y is important…

            No reasoning on earth can make that nature had a positive contribution in the past 55 years, with one exception (per Bart’s theory): if the natural cycle was extremely huge and fast, dwarfing the human contribution, the increase could be mainly natural.

            Not only violates that all available observations, but such a scenario implies that the natural cycle increased a fourfold in the past 55 years, as human emissions did increase a fourfold in that period and so did the yearly increase in the atmosphere and the net sink rate. Or you violate the equality of CO2, whatever the source, for the sinks.
            There is zero indication that the natural cycle increased over the past 55 years…

            I don’t think the pH trends are significant (yet), as the time series with sufficient precise measurements are too short and proxies are sparse and of lesser quality. But the trends in every place where is measured over the past decades are clear: all slightly down with increasing DIC, which proves that the net CO2 flux is from atmosphere into the ocean surface, not reverse…

            As there is a lot of evidence that the increase in the atmosphere is caused by humans, the increase in the oceans is caused by humans too. Especially if you look at the human “fingerprint” in the surface waters: the sharp decline in 13C/12C ratio, not seen in any ice core or sediments or coralline sponges or tree leaves over the past few million years…

            Accepting that humans are largely (> 90%) responsible for the increase of CO2 in the atmosphere and ocean surface doesn’t imply that one accepts that there is any danger connected to it: not in the atmosphere (warmer is better), not in the oceans as fish has no problems with much larger and faster pH changes and corals and coccoliths evolved in much higher CO2 levels in air and (warmer) waters…

          6. Bart

            Ferdinand – your continual insistence on the pathetically ill informed pseudo-mass balance argument shows that you just do not get dynamic systems, and you do not have the tools to understand this problem,

          7. Kenneth Richard

            Ferdinand: “Most people, myself included, have a tendency to uncritically accept what is said by others if that fits their own ideas, but I remain skeptic whatever the source…”

            Then explain your acute *lack* of skepticism when it comes to presuming that the IPCC’s stated values for natural CO2 sources and sinks are precise and accurate and therefore reliable. Obviously, you have no skepticism on this point. You’ve wholly accepted the IPCC values – even though they are little more than guesses. You, apparently, find these guesses to be wholly reliable. As a skeptic, I don’t find them reliable – especially since we seem to “discover” several new natural sources and sinks every year that we had no idea existed before. If we’re still in the discovery stage for natural sources and sinks, how can we (actually, you, since I’m not) be confident that the 118.7 GtC/yr and 78.4 GtC/yr (IPCC AR5) emitted by terrestrial respiration and the oceans (2013) are accurate figures?

            According to IPCC and Mauna Loa figures, natural sources of CO2 increased from a total 52.6 GtC/yr in 1970 to a total 198.1 GtC/yr in 2013, a difference of +145.5 GtC/yr. During this same period, human CO2 emissions increased from 4.3 GtC/yr (1970) to 8.9 GtC/yr (2013), a difference of +4.6 GtC/yr. In other words, according to the IPCC’s own figures and conversion formula, the natural sources of CO2 increased by nearly a factor of 4 between 1970 and 2013; anthropogenic sources increased by a factor of less than 2 during this same period. To uncritically accept what you do about these natural source figures requires you to also uncritically assume that natural *sinks* increased by identical amounts (and more). Where do you believe all these extra ~150 GtC/yr of natural sinks came from, Ferdinand? Did they exist in 1970? Or were they just not “discovered” yet? Can you understand why true skeptics like me might question why you believe what you do?

            Ferdinand: “As there is a lot of evidence that the increase in the atmosphere is caused by humans, the increase in the oceans is caused by humans too.”

            Using the IPCC values (again), the oceans absorbed 2.3 GtC of anthropogenic emissions in 2013 (26% of 8.9 GtC). They absorbed 80 GtC of natural CO2 emissions, a ratio of more than 30:1. In 1970, when the IPCC necessarily indicates the natural emissions CO2 sources total (land and oceans) amounted to 52.6 GtC/yr (as opposed to 198.1 GtC/yr total for 2013), the oceans only absorbed about 20 GtC, and humans contributed about 1 GtC (26% of 4.3 GtC). So why is it that you believe that the +60 GtC/yr increase in natural CO2 emissions absorbed by the oceans between 1970 and 2013 has had essentially no effect on the presumed change in the CO2 content in the oceans, but the extra ~1.3 GtC/yr of increases in human emissions from 1970 to 2013 has had nearly all to do with the CO2 content changes in the oceans?

          8. Ferdinand Engelbeen

            Bart,

            The mass balance must be obeyed at any moment of time and space. Except if CO2 escapes to space or is formed from cosmic rays, but that are extremely tiny amounts…

            Thus as human emissions are about twice as high as what is seen as increase in the atmosphere, either the natural cycle is rather modest and relative stable over time, or the natural cycle is enormous and fast, but must mimic human emissions in exact ratio over the past 55 years: a fourfold increase. No (theoretical) escape possible. There is zero evidence for such an increase in natural cycle speed…

            And Bart, I may not have the theoretical knowledge you have of high frequency dynamic processes, but you clearly lack the practical experience of what is possible and impossible in real life physical/chemical processes.

          9. Bart

            “The mass balance must be obeyed at any moment of time and space.”

            We’ve been over and over this. You are not doing a true mass balance. It is a pseudo-mass balance. This is a dynamic system. The sinks adjust to the inputs. Therefore, part of the sink activity is artificially induced. Indeed, these are for all intents and purposes artificial sinks.

            It does not matter if nature is a net sink overall. What matters is if nature would have been a net sink had there been no anthropogenic input. It would not. That makes nature a net source.

            I know you cannot understand this, because we’ve been over it time and time again. But, it is a very basic error on your part, and on the part of all the other jejune commenters who place stock in the pseudo-mass balance argument.

          10. Bart

            To do a true mass balance, you must break it down into natural and artificial sinks. Yes, there are artificial sinks, in direct proportion to artificial forcing. Something is acting up with my responses, some of which appear to be disappearing, but if previous one appears later, it will have more detail.

          11. Kenneth Richard

            Ferdinand: “No reasoning on earth can make that nature had a positive contribution in the past 55 years”

            Nature’s source contribution growth was +145.5 GtC/yr between 1970 and 2013. For humans, the growth was +4.6 GtC/yr between 1970 and 2013. Can you specifically identify where the presumed +150 to +155 GtC sink increase (“always” larger than sources per you) came from?

          12. Ferdinand Engelbeen

            Kenneth,

            Sorry, replied down before reading your reply here…

            About:
            “presuming that the IPCC’s stated values for natural CO2 sources and sinks are precise and accurate and therefore reliable”

            Please have a better look at what I wrote: I didn’t even use the IPCC estimates for any individual CO2 in or out flux. I only used the world estimates of fossil fuel emissions (not even the unreliable land use changes) which have a reasonable accuracy (as based on taxes, probably underestimated…) and accurate measurements of CO2 in the atmosphere. That is all you need to make a mass balance (except for Bart’s case).

            About the discovery of many unknown natural sources and sinks: that is a matter of ongoing research, which makes more and more progress over time to bring the individual fluxes in chart. But you are jumping to conclusions if you think that are additional sources. They are not, see further.
            Take e.g. soil + plant respiration: at full speed at night, increasing local CO2 levels to 500-600 ppmv under inversion. More than completely eaten away during the day by photosynthesis and the net result is slightly more sink than source over a full year. Only halve of these movements are reaching the bulk of the atmosphere over the seasons…

            There is a rough but simple test to know if the overall natural fluxes changed over time: the residence time. The residence time of any CO2 molecule in the atmosphere is the total amount of CO2 divided by the throughput. That is around 5 years (800 GtC mass, 150 GtC/year throughput). There are lots of estimates from years ago to recent of the residence time. If you split them in two periods and look at the average of both periods, then you see a slight increase in residence time: a rather constant throughput in an increasing mass of CO2 in the atmosphere…

            See: http://www.co2web.info/ESEF3VO2.pdf page 13

            Anyway all the time there was an in/out exchange of around 150 GtC/year by natural CO2 fluxes and slightly more sink than source in the past 55 years… Herewith Bart’s high mass / high speed mass balance is refuted too. A bidirectional exchange of ~150 GtC in natural CO2, mainly seasonal, is by far not sufficient to dwarf the one-way additional human CO2 of ~9 GtC/year.

            “They absorbed 80 GtC of natural CO2 emissions, a ratio of more than 30:1”

            Again, be careful with cycles. With the latest figures, the seasonal cycles are:
            Ocean surface: ~50 GtC in and out
            Biosphere: ~60 GtC in and out (part that reaches the bulk of the atmosphere)
            Non/less seasonal:
            Deep oceans: ~40 GtC/year between upwelling and downwelling zones

            Biosphere and ocean surface act opposite to each other with seasonal temperatures and NH and SH also opposite to each other. The net effect is ~10 GtC change in de atmosphere, dominated by (NH) vegetation or globally ~5 ppmv over the seasons, mainly visible in the NH where the bulk of the biosphere is.

            What is released or absorbed by oceans and biosphere over the seasons is of no interest for longer term changes, only what remains after a full seasonal cycle is important. There is hardly any change in the amplitude of the seasonal cycle if you compare earlier periods with more recent: only a small increase in net result, about halve human emissions…

          13. Kenneth Richard

            Ferdinand: “Anyway all the time there was an in/out exchange of around 150 GtC/year by natural CO2 fluxes and slightly more sink than source in the past 55 years”

            If the “all the time” in/out natural exchange is ~150 GtC/yr, and the 2013 natural CO2 sources was 197.1 GtC/yr per the IPCC (118.7 terrestrial respiration, 78.4 ocean outgassing), with natural sinks slightly greater (necessarily over 200 GtC/yr), whereas the human input was 8.9 GtC/yr in 2013, that means that the natural input/output has increased by about 50 GtC since whenever you’re claiming the “all the time” ~150 GtC/yr rate was naturally occurring. So that’s…

            Nature (sources + sinks) +50 GtC/yr
            Human +9 GtC/yr

            …since the ~150 GtC/yr rate was breached – using your stated values. Humans have therefore contributed less than 20% to the increase in atmospheric concentration – again using your stated values and the IPCC’s figures. Where did those extra ~50 GtC/yr of natural sources and sinks come from, Ferdinand?

            It’s even worse than this for you, though. Using the accepted IPCC conversion factor 2.12 GtC = 1 ppmv CO2 and the Mauna Loa CO2 ppm values for 1970 and 2013 (per NOAA.gov), we learn that the change, or increase, in all CO2 carbon for 1970 to 2013 was +150.1 GtC (396.48 ppm [2013] – 325.68 ppm [1970] = 70.8 ppm x 2.12 GtC = 150.1 GtC).

            150.1 GtC was the total CO2 carbon increase from 1970 to 2013 using the IPCC formula. Of that increase, 4.6 GtC/yr was from anthropogenic emissions since 1970 (4.3 GtC/yr) to 2013 (8.9 GtC/yr).

            In total, then, the difference between 1970 natural emissions and 2013 natural emissions is +145.5 GtC/yr, which is 150.1 GtC/yr (natural total increase) – 4.6 GtC total increase from humans.

            So not only do you have to explain where the extra +50 GtC/yr from natural sources/sinks came from since the “all the time” rate of ~150 GtC/yr was crossed (it was 197.1 GtC/yr total from nature for 2013), it looks like you’re also going to have to explain where the +145.5 GtC/yr of natural sources/sinks has come from since 1970 to explain how we got from 325 ppm to 396 ppm using the IPCC’s conversion formula.

            Let’s see what you come up with.

            So not only do you have to explain where the extra +50 GtC/yr from natural sources/sinks came from since the “all the time” rate of ~150 GtC/yr was crossed per the IPCC’s stated values, it looks like you’re also going to have to explain where the natural +145.5 GtC/yr of natural sources/sinks has come from since 1970.

          14. Ferdinand Engelbeen

            Kenneth:

            “the 2013 natural CO2 sources was 197.1 GtC/yr per the IPCC (118.7 terrestrial respiration, 78.4 ocean out gassing)”

            You forget that not all of these fluxes reach the bulk of the atmosphere: much of soil respiration and nighttime leave respiration is captured the same day in sunlight by photosynthesis.

            And again I am not responsible for what the IPCC says or thinks or calculates.

            Moreover:

            “that means that the natural input/output has increased by about 50 GtC” can’t be right. Either the natural input/output was around 200 GtC all the time, which means a residence time of 4 years or there must have been a decrease in residence time over time, while the opposite is seen in the estimates.

            A change in residence time over the past 55 years conflicts with the atomic bomb tests 14C spike, which shows a rather constant decay rate, no acceleration. The same for the 13C/12C decrease: the same “thinning” of the human fingerprint over time, no deceleration if there was an increase in CO2 turnover.

            “Nature (sources + sinks) +50 GtC/yr
            Human +9 GtC/yr”

            Sorry Kenneth, you are completely lost here: even if we assume that the natural sources increased with 50 GtC over the period 1970-2013, the natural sinks increased with more than 50 GtC, as in every year the natural balance was negative. That gives:

            Nature: sources + 50 GtC/yr – sinks – 52.5 GtC/yr
            Humans +9 GtC/yr

            Net result in 2013 compared to 1970:
            increase = 9 GtC + 150 + 50 – 152 – 52.5 = 4.5 GtC
            or ~2.15 ppmv/year
            With an increased turnover of ~200 GtC/year and a resulting residence time for any CO2 molecule of ~4 years (that is less than estimated from different observations).

            “to explain how we got from 325 ppm to 396 ppm”

            Not that difficult: humans emitted 275 GtC in the period 1970-2013 or about 130 ppmv, by far enough to explain the 71 ppmv increase in the atmosphere…

          15. Ferdinand Engelbeen

            Bart,

            Some follow up from a lost comment up here…

            “The sinks adjust to the inputs. Therefore, part of the sink activity is artificially induced.”

            Bart you make a basic error: the sinks don’t adjust to the inputs, the sinks adjust to the total increase in the atmosphere, not the momentary input of one year. Moreover, different sinks adjust to different causes: some sinks (and sources) react mainly on temperature, others mainly on pressure and some on both.

            “What matters is if nature would have been a net sink had there been no anthropogenic input. It would not. That makes nature a net source.”

            That is one big jump to the conclusion that your theory is right…

            If there were no human emissions, nature was neither sink or source at constant temperature, it would be at steady state. Temperature changed the steady state with about 16 ppmv/°C over the past 800,000 years as seen in ice cores and in accordance with Henry’s law.

            As we are currently some 110 ppmv above steady state, stopping all human emissions today still would show a net sink of ~2.15 ppmv/year the first year after the stop, gradually declining to zero when the steady state for the current (area weighted) average temperature is reached again. With an e-fold decay rate of over 50 years…

          16. Kenneth Richard

            Ferdinand: “Either the natural input/output was around 200 GtC all the time, which means a residence time of 4 years or there must have been a decrease in residence time over time, while the opposite is seen in the estimates.”

            Elsewhere you write that the “all the time” natural source/sink yearly rate is ~150 GtC/yr. Now you’re claiming it’s 200 GtC/yr. Have you decided which one is the correct “all the time” rate? If the “all the time” rate is now 200 GtC/yr, and it used to be 150 GtC/yr, where did that extra 50 GtC/yr come from? When did the rise from 150 GtC/yr to 200 GtC/yr start?

            “And again I am not responsible for what the IPCC says or thinks or calculates.”

            I haven’t said you are responsible. The IPCC has the yearly natural rate at 197.1 GtC (2013). You claim the “all the time” rate is 150 GtC. The IPCC also clearly uses a formula that indicates that the rise from 325 ppm to 396 ppm, the natural sources/sinks increased by +145.5 GtC/yr between 1970 and 2013, whereas the anthropogenic contribution only increased by +4.6 GtC/yr during that span. Are you claiming the scientists who developed the formula are wrong? Can you explain what source you are using to say it’s wrong, or is this just your opinion?

            https://www.ipcc.ch/publications_and_data/ar4/wg1/en/ch7s7-3-1-3.html
            “…using a conversion factor of 2.12 GtC yr–1 = 1 ppm.”

            (396.48 ppm [2013] – 325.68 ppm [1970] = 70.8 ppm x 2.12 GtC = 150.1 GtC/yr – 4.6 GtC/yr [anthro] = +145.5 GtC/yr [natural] for 1970-2013)

            Ferdinand: “even if we assume that the natural sources increased with 50 GtC over the period 1970-2013, the natural sinks increased with more than 50 GtC”

            And this is your belief, of course, because you presuppose that anthropogenic emissions are the cause of all Mauna Loa increases. And because you presuppose that, therefore, no matter what the increase or change in natural sources, the sinks are automatically believed to have increased by slightly more. That way the belief that you have remains intact. It’s an *unfalsifiable* belief. Do you have any idea where these new sources or sinks are coming from? Of course you don’t. But you believe in them nonetheless. A skeptic you are not.

            Ferdinand: “humans emitted 275 GtC in the period 1970-2013 or about 130 ppmv, by far enough to explain the 71 ppmv increase in the atmosphere…”

            Natural emissions were about 25 to 35 times greater than the human output between 1970 and 2013. In fact, in the year 2013 alone, the natural output was 197.1 GtC (per the IPCC), which is 70% of the output for the entire 44 years from anthropogenic sources. But, of course, you’ll say that these massive emissions have been carefully and categorically distinguished from the anthropogenic emissions in the atmosphere, such that *only* the natural emissions are offset by sinks, and the anthropogenic emissions can therefore remain unaccounted for, and thus your belief remains intact that anthropogenic emissions are what cause changes in atmospheric CO2. The unfalsifiable presupposition remains intact, making you right no matter what.

  11. Peter

    Error in previous post

    “Causing co2 to rise” should be “causing ph to rise”

  12. yonason

    Back in 2011 Judith Curry said 75%, while this geologist said 95%.

    Looks like a difference of opinion to me.

    I don’t know why Willis gets so hysterical whenever anyone says something he disagrees with. He alienates a lot of people. If he were the only other guy on my team, I’d quit the team.

    1. yonason
    2. Kenneth Richard

      I recently replied to one of the comments in a WUWT Willis essay claiming that the Sun/CRs have little to no effect on climate by pointing out that there have been over 200 papers published in the last 27 months (through late March, with 15 more added since then) linking the Sun to the Earth’s climate. His response? He insults Pierre Gosselin, calls papers that link the Sun to climate “garbage” and “a piece of trash,” and tells me that providing such (NTZ) links to the paper lists is “handwaving.”

      https://wattsupwiththat.com/2016/04/09/cosmic-disconnections/

      Mine is the 4th comment down (“kennethrichards”). Willis’ initial reply is the 5th. There are several more exchanges afterward. And others much further down in the thread too.

      Bottom line, Willis’ I’m-Right-So-If-You-Disagree-With-Me-You’re-Lying comment in this thread is similar to how he behaves on WUWT when someone writes something that challenges his beliefs or perceptions of “Truth.” A style that, in my view, is the opposite of what would be expected from someone who claims to be a scientist.

      1. yonason

        “There are several more exchanges afterward. And others much further down in the thread too.” – Kenneth Richard

        I’m surprised the concrete thinking blockhead moderators didn’t shut you down. After the last time that happened to me I banned myself from the website.
        – – – – – – – – –

        Here’s something by some guy named Landscheidt.
        http://www.john-daly.com/solar/solar.htm

        I wonder if Willis has ever heard of his “garbage.”
        – – – – – – – – –

        Nice work digging up all those papers, btw.

      2. yonason

        P.S. – After Willis called Nir Shaviv a liar, Shaviv gave Watts and Co., the old heave ho.
        https://disqus.com/by/nirshaviv/
        Can’t say I blame him, either.

  13. Ferdinand Engelbeen

    Well, I listened to the different lectures of Dr. Salby on the net and was there at his presentation in London a few years ago. I posed several questions to him after the lecture which were all evaded and couldn’t have more discussion as I was not properly dressed (no tie!) to follow the orginisation into the catacombes of the Parliament…

    I haven’t read his book, as I suppose that the physics will be right and that Pierre has extracted the most interesting points, which are where I have some knowledge of:

    a. CO2 follows temperature changes:
    That still is true, but limited: historically over the past 800,000 years a ratio of ~16 ppmv/°C with a lag of 800 +/- 600 years after temperature. Seasonally ~5 ppmv/°C and year by year variability (El Niño, Pinatubo) 4-5 ppmv/°C. That explains about 15 ppmv from the 110 ppmv increase since the LIA, the rest is not caused by temperature.

    b. CO2 is a small part of the greenhouse effect:
    Willis is right, based on very accurate line by line measurements in labs (Hitran project of the US military) at different pressures of different mixings of air, water vapor, CO2 and CH4, CO2 at the current mix makes about 20% of the greenhouse effect. Not negligible. A doubling of CO2 makes a difference of ~1°C at ground level. Quite benign in effect, no catastrofic effects in sight…

    c. paleoclimate CO2 concentrations are probably inaccurate (too low and too stable):
    Depends of the resolution of the ice cores, which gets worse back in time: from 10 years in the past 150 years to 560 years over the past 800,000 years. Despite that, the current increase over the past 160 years would be measurable in all ice cores, even with the worst resolution, be it with a lower amplitude.
    Too low is near impossible. I heard him say in his Hamburg lecture (on the net) that the interglacial peaks may be underestimated with a factor 10, or might have been 3000 ppmv CO2 i.s.o. 300 ppmv. As thus 2700 ppmv may have spread by (impossible) diffusion over the 90% time of the glacial periods which show ~180 ppmv, the latter thus had originally negative CO2 values, eventually killing all (plant) life on earth… Even a lower underestimate is near impossible, as 180 ppmv is already the lowest border for the survival of C3 plants…

    More to come…

    1. AndyG55

      “Ferdinand Engelbeen: “Willis is right … CO2 at the current mix makes about 20% of the greenhouse effect”

      Mr Luke Warmer talks RUBBISH again.

      CO2 has basically ZERO greenhouse effect, because it DOES NOT effect convection.

      1. Ferdinand Engelbeen

        AndyG55, you may believe that yourself, but radiation physics show (proven by Tyndall over 100 years ago) that IR absorption + sufficient chance of collisions with other molecules (O2, N2) warms the local atmosphere. I suppose that does affect convection, but the theoretical and practical consequences I have learned too long ago for my memory…
        See for an in depth explanation of the greenhouse effect:

        http://www.warwickhughes.com/papers/barrett_ee05.pdf

        About the percentages:
        http://www.barrettbellamyclimate.com/page23.htm

        Dr. Jack Barett is a known (luke-warmer) skeptic as I am and Prof. David Bellamy was banned from the BBC for his skeptic stand…

        1. AndyG55

          Will be funny watching you futz about when the temperature start dropping over the next decade or so.

          CO2.. BULLSHIT !!

        2. AndyG55

          “too long ago for my memory…”

          yep..old “settled science” roflmao !!

        3. AndyG55

          “proven by Tyndall over 100 years ago”

          Tyndall worked with enclose gases.

          The atmosphere is NOT enclosed gases, they are open to real convection.

          The cases are totally different.

          In the lower atmosphere, convection rules.. any tiny warming from extra CO2 absorption, which exhausts in the very lowest part of the atmosphere, passes energy to other molecules and is immediately counteracted by convection.

          1. Ferdinand Engelbeen

            AndyG55,

            No matter the end result, the fact that CO2 absorbs heat at a certain frequency and in part re-emits it, in part redistribute it to other molecules by collisions, makes it a greenhouse gas.

            As far as I remember, the ratio between re-emission and collision at 1 bar (ground level) is about 1:1 and of course gets lower with height.

            That much of the heat is redistributed by convection doesn’t change that fact, only that convection may redistribute the warming towards the poles and acts as a negative feedback. Which doesn’t imply zero net effect…

          2. AndyG55

            CO2 does very little re-emitting below about 11km. It does not warm the atmosphere.

            Will Happer showed that the re-emission time is several orders higher than collisional time in the lower atmosphere. You have your collision to re-emit rate very wrong.

            There is no mechanism for CO2 warming in the atmosphere.

          3. AndyG55

            With CO2 a “well mixed gas”,

            There has been no warming in the Antarctic for 37+ years of the satellite record.

            No warming in the Southern Ex tropics in 20 years.

            The ONLY warming in the satellite record is from El Nino events, which have affected mostly the NH.

            CO2 is a absolute non-starter.

            Doesn’t event make it to the stalls.

          4. Ferdinand Engelbeen

            AndyG55,

            Indeed I was wrong about the emissions:collision ratio, but it is the opposite of what you say…

            Thus still the collissions heat the local atmosphere, additionally to the already present convection. That may increase convection or may influence the lapse rate. In the latter case, the surface temperature goes up…

          5. AndyG55

            “or may influence the lapse rate”

            OMG, what a stupid statement. I thought you “knew” stuff.

            Seems I was wrong.. you seem basically ignorant about some things..

            The only thing that influences the lapse rate is H2O, because it is the only substance to effectively change the specific energy of the atmosphere.

            That is why the dry lapse rate is around 9.8ºC/km and the wet lapse rate is around 6.5ºC/km.

            CO2 has an immeasurable effect on the lapse rate. (increasing it by a miniscule amount, actually)

          6. AndyG55

            nb, wet lapse rate can drop as low as 5ºC/km for saturated air.

            Convection is just another conduit for cooling. The conduits work together, but convection and conduction rule in the lower atmosphere.

            There is no mechanism that allows for warming from increased atmospheric CO2.

          7. DirkH

            AndyG55 15. May 2016 at 10:27 PM | Permalink
            “CO2 does very little re-emitting below about 11km.”

            Correct.

            ” It does not warm the atmosphere.”

            Wrong.

            What Co2 does not re-emit it keeps as vibrational energy and transfers to neighbouring molecules. This vibrational energy is commonly known as “heat”.

            It is exactly the non-re-emission that constitutes the local warming.(Which is then transported upwards by convection, in my opinion)

          8. AndyG55

            No Dirk,

            It will only warm the atmosphere if it is not counteracted by convection, and it will be.

            A small increase in CO2 will marginally change the height of complete IR absorption, no more effect that striking a match or having a bushfire. The heat is gone quickly.

        4. yonason

          “IR absorption + sufficient chance of collisions with other molecules (O2, N2) warms the local atmosphere.” – Ferdinand Engelbeen

          Any chance you could give not only the reference, but a specific “money quote” for that? …because there’s not a lot of evidence I’ve seen for that claim, and going on a snark hunt isn’t my idea of fun. From what I’ve found, it’s the contact of the atmosphere with the ground at a higher temp that warms the air. The first demonstration of that being the Robert Wood experiment.

          Thanks in advance

          1. Ferdinand Engelbeen

            yonason,

            In my memory the ratio between re-emission and collision for CO2 at 1 bar air pressure was about 1:1, but my memory was completely wrong (getting worse with getting older…). It seems more like 1:10,000. See the work of Jeff Id (frequent commenter at Climate Audit):

            https://noconsensus.wordpress.com/2010/08/17/molecular-radiation-and-collisional-lifetime/

            I can’t follow everything as most is way over my head, but I have the impression that radiation still largely goes at the same speed, as collisions work bidirectional and may exite a CO2 molecule which re-emits a photon, despite that huge ratio… The latter is used to excite CO2 in CO2-lasers.

            From the other side of the fence (Eli Rabett) a similar explanation:
            http://rabett.blogspot.be/2013/04/this-is-where-eli-came-in.html

            That is independent of convection, which probably makes the bulk of the heat transfer in the atmosphere.

          2. DirkH

            After thermalization, (energy transfer from photon to vibrational energy of CO2 molecule), one of two things can happen
            -dethermalization – re-emitting the photon
            -collision with another molecule, transferring the vibrational energy

            Dethermalization happens very quickly so it depends on how long it takes to collide with another molecule whether the 2nd possibility is taken.
            To warm the local atmosphere you need to have this, so that the energy goes to non-IR-emitting biatomic molecules like O2.

            So under a certain height, warming the atmosphere SHOULD happen, over that height, the opposite happens, and CO2 becomes the transmitter of energy to space via dethermalization.

            Below that critical height, thermalization happens more often, above it, dethermalization.
            Below it, CO2 WARMS, above it, it COOLS.

            Having said all that, there is large scale vertical transport of heat by convection which has nothing at all to do with radiation. The warmed local atmosphere will MOVE upward (and expand, and cool, and reach the height where dethermalization and radiation to space dominate)

            So whether or not “Greenhouse effect” happens looks a bit more complicated than using MODTRAN – which only simulates the radiative aspects – not the thermalization/dethermalization/convection/conduction.

          3. DirkH

            …the models predict a hot spot in the troposphere (through the thermalization of IR photons). This has not been observed by weather balloons or satellites. So the models are broken. They likely do not simulate changes in convection correctly.

            That’s why every climate scientist coming out with warming predictions is an irresponsible propagandist. They know that their models are broken yet they sound the alarm, to ensure their further funding.

            Like welfare queens only a million times more expensive.

          4. Ferdinand Engelbeen

            DirkH,

            Two questions, as my knowledge on that aspect of climate is not that good:

            1. Is the alleged “hot spot” not more a matter of water vapor, alleged to increase with higher temperatures, but not seen in the measurements?

            2. As CO2 is quite well mixed, the atmosphere is quite evenly warmed up by the extra CO2, only dependent of the radiation from the surface. For relative small areas that doesn’t make much difference in convection compared to the already existing convection caused by temperature (and humidity) differences at ground level I suppose, making that radiation is mostly additive to the warming?

          5. DirkH

            “2. As CO2 is quite well mixed, the atmosphere is quite evenly warmed up by the extra CO2, only dependent of the radiation from the surface.”

            It is also dependent on local pressure. Higher pressure means quicker collisions, therefore, thermalization happens more often than spontaneous re-emission. Under lower pressure, the molecule is more likely to re-emit, before it can collide with another molecule.

            This changes the lapse rate, and as far as I understand, that is what causes the Hot Spot in the first place – which is “hot” insofar as it is “hotter than in the absence of increased CO2”.

            The lapse rate change is a dent; it is nonlinear – below critical height, cooling is delayed, above it, it is accelerated.

          6. DirkH

            Instead of
            ” that is what causes the Hot Spot in the first place ”
            i should say
            ” that is what is supposed to cause the Hot Spot in the first place ”
            because – it isn’t observed.

            The radiative physics are likely correct, it must be convection or something else that prevents the hot spot from actually materialising.

          7. yonason

            @ DirkH

            From what I’ve read, you’re spot on. Nice job!

          8. yonason

            @Ferdinand Engelbeen 15. May 2016 at 6:18 PM

            Are you under the impression that you understand what he said? …because I do not.

            “In inelastic collisions, kinetic energy is converted to vibrational energy and back. Most molecular collisions are elastic and total kinetic energy is preserved. Only about 1 in 10,000 collisions is inelastic at Earth surface temperature and pressure.”

            so far so good, but then he writes…

            “This also means that only about 1 in 10,000 excited molecules decays by emission of radiation rather than collision.”

            So, in the first part he tells us that 9,999 collisions exchange no potential energy (decay), i.e., they are elastic, but that one is inelastic (exchanges potential energy); but then in the second part we are told that in 9,999 cases there is inelastic collision, with one molecule decaying by emission rather than collision.

            I may be missing something, but if I haven’t, then that makes no sense, unless he’s referring to two different processes, but if he is it isn’t clear to me.

            Any thoughts?

          9. Ferdinand Engelbeen

            yonason,

            As far as I understand – not the best of my knowledge – the elastic collisions still remove energy from the exited molecule, thus removing the possibility of emitting a photon (but still total energy is preserved, be it redistributed over the molecules). An inelastic collision is biderictional and a sufficient energetic O2 or N2 molecule can exite a CO2 molecule. That makes that anyway 1:10,000 excited CO2 molecule will emit a photon, but not necessary one of the molecules that were excited in the first place…

            But I may be completely wrong…

    2. AndyG55

      “A doubling of CO2 makes a difference of ~1°C at ground level”

      Immediately counteracted by natural convection.

    3. Bart

      http://woodfortrees.org/plot/esrl-co2/derivative/mean:12/from:1979/plot/rss/offset:0.6/scale:0.22

      Still holding. You are wrong, Ferdinand. It is not a simply proportionality of total concentration but in the rate of change. The sensitivity is in ppmv/K/unit-of-time.

      1. Ferdinand Engelbeen

        Bart,

        The absorption of the sinks, mainly the oceans, is directly proportional to the extra pressure of CO2 in the atmosphere above (dynamic) equilibrium. That is elementary process dynamics, surprisingly linear over the past 55 years. Temperature only modulates the equilibrium with 16 ppmv/K that is all. The steady state level for the current (area weighted) average temperature is 290 ppmv, not 400 ppmv.

        Moreover, most of the variability is the response of vegetation to temperature, which zeroes after 1-3 years. Its long time effect is negative, thus not the cause of the increase.

        The sensitivity is ppmv/K whatever time is needed to reach that value. Any increase in temperature is fully compensated with an increase in pCO2 in the atmosphere at 16 ppmv/K. Then it stops.

        1. Bart

          “The absorption of the sinks, mainly the oceans, is directly proportional to the extra pressure of CO2 in the atmosphere above (dynamic) equilibrium.”

          At any given time. But, the oceans are flowing, with currents drawn into the depths, and sent back up again over a timespan of centuries. The flow is modulated by temperature, hence the Thermo-Haline Circulation. This naturally begets a temperature dependent rate of change of CO2.

          1. Ferdinand Engelbeen

            Bart,

            The THC circulation may be influenced by temperature, there is indeed a seasonal component in it, but any trend on long term is mere speculation from science fiction films like “The Day after Tomorrow”. Anyway, warmer ocean waters at the sink places would reduce the sink branch of the THC which mainly depends of ice formation, which increases the salt content and thus density of the remaining waters. Thus if there is any influence, warmer means less circulation and thus less CO2 release at the upwelling places and less CO2 uptake at the sink places. Net effect probably around zero, as the “less” is at both sides of the equation…

            Thus there is probably little influence of the THC strength on CO2 levels in the atmosphere, even less in the rate of change and if there is, opposite to what you expect (or hope…).

            It is thought (Mõrner) that during the LIA, the THC was much weaker, in part deflected towards North Africa. That resulted in colder temperatures in NW Europe and a drop of ~6 ppmv CO2. That was all.

          2. Bart

            There cannot be less CO2 release at the upwelling places. That CO2 was carried down to the depths centuries ago, and is now rising again. Stopping it is like stopping a diesel locomotive with a thousand car train, only more so. The inertia is immense, and stretches all the way down the current stream.

          3. Ferdinand Engelbeen

            Bart,

            If there were no exchanges with the atmosphere, all CO2 content of the upwelling waters is simply transported to the ocean water sinks.

            The CO2 fluxes between ocean surface waters and atmosphere are directly proportional to the pCO2 pressure difference between water and atmosphere.

            If the pCO2 pressure in the oceans goes up (by temperature or concentration), the pCO2 difference is increased and so is the influx into the atmosphere.

            If the pCO2 (~ppmv) of the atmosphere increases for whatever reason, the pCO2 difference with the upwelling waters is decreased and so is the influx into the atmosphere. What is not transported into the atmosphere simply remains in the water parcel.

            The same parcel of water may start as a huge source of CO2 at the upwelling place, a moderate source along the equator, neutral when the temperature has sufficiently dropped to equal pCO2 of water with the atmosphere, a moderate sink when getting colder on its trip to the poles and a huge sink at the edge of the ice fields…

  14. Ferdinand Engelbeen

    Next comments:

    Page 546: “However, because they are so much stronger, even a minor imbalance between natural sources and sinks can overshadow the anthropogenic component of CO2 emission.”:

    “Can” overshadow, but don’t. simply look at the year by year variability of the natural balance: maximum +/- 1.5 ppmv around the trend of +80 ppmv for the most extreme changes in temperature (Pinatubo, El Niño):

    http://www.ferdinand-engelbeen.be/klimaat/klim_img/dco2_em2.jpg

    Human emissions are twice as large…

    Page 249/250: “These and other processes that control radiatively active species are temperature dependent.”

    Yes, but not dominant in the past 160 years: the maximum response of the oceans to temperature (per Henry’s law) in dynamic equilibrium (“steady state”) is ~16 ppmv/°C. Henry’s law gives 4-17 ppmv/°C in the literature. That is all. Higher temperatures give more decomposition of organic matter, but also more growth. Growth largely wins from decay with higher temperatures over the seasons and slightly wins over longer (> 1-3 years) periods:

    http://www.bowdoin.edu/~mbattle/papers_posters_and_talks/BenderGBC2005.pdf

    Page 253: “they imply that a significant portion of the observed increase in r˙CO2 derives from a gradual increase in surface temperature.”

    Here he makes the same mistake as many before him: the high correlation between temperature variability and CO2 rate of change variability says next to nothing about the cause of the slope in the rate of change. In fact, the opposite δ13C and CO2 changes prove that the CO2 variability is caused by the response of vegetation to temperature variability, but vegetation is a net, growing sink for CO2 over time, thus not the cause of the increase… Neither are the oceans: the temperature increase 1960-2015 is 0.4°C, good for 6 ppmv from the 80 ppmv increase in that period.

    Page 254: “The two proxies of previous climate are incompatible. They cannot both be correct.”

    With an established sensitivity of 16 ppmv/°C, following Henry’s law, both are correct. Only if you assume a much larger sensitivity of CO2 for temperature changes, both are incompatible. I think that Dr. Salby is wrong on this point (and several other points)…

    That humans are responsible for the current increase in CO2 is based on all available evidence. All alternative explanations I heard of (like temperature) violate one or more observations. See further:

    http://www.ferdinand-engelbeen.be/klimaat/co2_origin.html

    Which doesn’t imply that more CO2 isn’t better for plant life or has any catastrophic consequences…

    1. Kenneth Richard

      Ferdinand: “That humans are responsible for the current increase in CO2 is based on all available evidence.”

      Really? All available evidence? Then can you explain why it is that the airborne fraction (proportion/ratio from fossil fuels) has been essentially constant in the 50 years, or even since industrialization began (see the papers below)? Why wouldn’t there be at least a *modest* correlation between the explosion in fossil fuel consumption and the airborne fraction?

      Here’s what that non-correlation looks like over the last ~50 years:

      http://ej.iop.org/images/1748-9326/8/1/011006/erl459410f3_online.jpg


      http://www.atmos-chem-phys-discuss.net/10/9045/2010/acpd-10-9045-2010.html
      The ratio of CO2 accumulating in the atmosphere to the CO2 flux into the atmosphere due to human activity, the airborne fraction (AF), is central to predict changes in earth’s surface temperature due to greenhouse gas induced warming. This ratio has remained remarkably constant in the past five decades

      http://onlinelibrary.wiley.com/doi/10.1029/2009GL040613/full
      Abstract: [T]he trend in the airborne fraction since 1850 has been 0.7 ± 1.4% per decade, i.e. close to and not significantly different from zero. The analysis further shows that the statistical model of a constant airborne fraction agrees best with the available data if emissions from land use change are scaled down to 82% or less of their original estimates. Despite the predictions of coupled climate-carbon cycle models, no trend in the airborne fraction can be found.

      Conclusion: From what we understand about the underlying processes, uptake of atmospheric CO2 should react not to a change in emissions, but to a change in concentrations. A further analysis of the likely contributing processes is necessary in order to establish the reasons for a near-constant AF [ratio of CO2 accumulating in the atmosphere to the CO2 flux into the atmosphere due to human activity] since the start of industrialization. The hypothesis of a recent or secular trend in the AF cannot be supported on the basis of the available data and its accuracy.

      In addition to explaining the above airborne fraction non-correlation, I’m curious if, in the consideration of “all” the evidence, the conclusions from the below papers have been included in that considered “evidence.” (There are several more papers like this, but this post is going to be too long as it is.) From reading these paper summaries alone, it doesn’t appear that all evidence has been considered in reaching the conclusion that the increase in CO2 is almost exclusively due to fossil fuels.

      http://eae.sagepub.com/content/20/7/1149.abstract
      It is commonly assumed (e.g. by the UN Intergovernmental Panel on Climate Change; IPCC) that a part of the emitted carbon dioxide will stay in the atmosphere and, therefore, large emission rate of carbon dioxide should cause a large increase rate of atmospheric carbon dioxide. High temperature should also increase the atmospheric carbon dioxide concentration due to lowered solubility of carbon dioxide in the backmixed ocean surface water. However, using two-dimensional regression analysis, the increase rate could not be explained by the emissions because temperature was the dominating parameter that controlled the increase rate. The fraction of the emissions that remained in the atmosphere—or the airborne fraction—decreased significantly despite global warming

      http://eae.sagepub.com/content/20/1/105.abstract
      The results suggest that El Nino and the Southern Oscillation events produce major changes in the carbon isotope ratio in the atmosphere. This does not favour the continuous increase of CO2 from the use of fossil fuels as the source of isotope ratio changes. The constancy of seasonal variations in CO2 and the lack of time delays between the hemispheres suggest that fossil fuel derived CO2 is almost totally absorbed locally in the year it is emitted. This implies that natural variability of the climate is the prime cause of increasing CO2, not the emissions of CO2 from the use of fossil fuels.

      http://www.researchgate.net/publication/281111296_RESPONSIVENESS_OF_ATMOSPHERIC_CO2_TO_ANTHROPOGENIC_EMISSIONS_A_NOTE
      A statistically significant correlation between annual anthropogenic CO2 emissions and the annual rate of accumulation of CO2 in the atmosphere over a 53-year sample period from 1959-2011 is likely to be spurious because it vanishes when the two series are detrended. The results do not indicate a measurable year to year effect of annual anthropogenic emissions on the annual rate of CO2 accumulation in the atmosphere.

      Also, the official CO2 record shows that, during the 13-year period from 1938 to 1950, atmospheric CO2 remained constant at 311 ppm. Can you explain why the atmospheric CO2 concentration remained the same during that period in light of your conclusion that only anthropogenic emissions can explain CO2 concentration changes?

      http://i90.photobucket.com/albums/k247/dhm1353/Law17501875.png
      The opposite of the above question applies too. Looking at this Law Dome graph, notice that CO2 increased from ~277 ppm to ~290 ppm from 1750 to 1875 while changes in CO2 emission from humans were effectively non-existent. How do you explain that discrepancy?

      1. Ferdinand Engelbeen

        Kenneth: “Why wouldn’t there be at least a *modest* correlation between the explosion in fossil fuel consumption and the airborne fraction?”

        Because there is no reason at all that there should be a correlation. The net sink rate is a function of the extra CO2 pressure in the atmosphere above steady state, not the momentary emissions, and the steady state is influenced by temperature. The airborne fraction then is the net difference of the yearly emission and the net sink rate of that year.

        That it is around 50% of the emissions is pure coincidence of two factors: a slightly quadratic increase of human emissions over time, followed by a slightly quadratic increase in the atmosphere and thus net sink rate. And a rather long decay rate of excess CO2 to again reach the steady state (~51 years). That gives a rather constant ratio between emissions and increase in the atmosphere:
        http://www.ferdinand-engelbeen.be/klimaat/klim_img/temp_emiss_increase.jpg
        If human emissions halved today, sinks would be constant and the increase in the atmosphere got zero. If emissions ceased totally, the sinks would be the same as today the first year, gradually decreasing to zero when the steady state is reached again…

        From Dr. Ahlbeck:
        “However, using two-dimensional regression analysis, the increase rate could not be explained by the emissions because temperature was the dominating parameter that controlled the increase rate.”

        Common error: there is a high correlation between the temperature rate of change and the CO2 rate of change, Thus the trend in the CO2 rate of change is caused by the trend in temperature…

        Problem is that most of the variability in CO2 rate of change is the response of (tropical) vegetation to ocean temperatures (El Niño) but vegetation shows a negative trend in CO2 with temperature over longer periods (> 1-3 years). Ocean temperature is only a small player here.

        The work of Tom Quirk was firmly criticized by Jack Barret and me in a response published in E&E (Comments on Sources & Sinks of Carbon Dioxide by Tom Quirk, E & E, 20, 103, (2009)):

        Quirk assumes no time delay between the NH and SH by comparing the CO2 changes over the seasons, but he forgot that a lag (or lead) of 12, 24, 36 months gives exactly the same CO2 changes over the seasons. There is certainly a lag of the SH after the NH, both for CO2 and δ13C:
        http://www.ferdinand-engelbeen.be/klimaat/klim_img/co2_trends_1995_2004.jpg
        and
        http://www.ferdinand-engelbeen.be/klimaat/klim_img/d13c_trends.jpg
        Which proves that the emissions of extra low-13C CO2 are in the NH.

        “is likely to be spurious because it vanishes when the two series are detrended”

        I have tried to explain to Munshi – to no avail – that by detrending he is exactly removing the real correlation and only looking at the remaining variability…
        If you add a nice sinusoid without a trend to a completely linear trend, the resulting sum is a nice sinusoid with a trend. If you detrend the latter you only have left the sinusoid without a trend and there is of course no correlation left with the original trend…

        “Can you explain why the atmospheric CO2 concentration remained the same during that period”

        Between 1938 and 1950 the temperature dropped with 0.2°C, good for a drop of 3 ppmv CO2. Thus the emissions were good for an increase of 3 ppmv in that period. Human emissions in the same period were about 10 ppmv. Looks good to me…

        For the period 1750-1875, no idea, natural variability, increased land use change (wood burning without replanting, increased CH4 by rice padding). The industrial revolution did only break loose from 1850 on…

        I didn’t say that temperature has no influence, but it is far less than expected by some here (about 10% of the total increase since 1850). Temperature still is responsible for almost all of the variability in rate of change of the CO2 increase.

        1. Kenneth Richard

          “Because there is no reason at all that there should be a correlation.”

          So we don’t even need to have a correlation to nonetheless secure 100% causation? Interesting. So why do scientists (even James Hansen!) express something close to bewilderment that the airborne fraction has remained steady, even decreased, as human emissions have grown exponentially?

          “However, it is the dependence of the airborne fraction on fossil fuel emission rate that makes the post-2000 downturn of the airborne fraction particularly striking. The change of emission rate in 2000 from 1.5% yr-1 to 3.1% yr-1 (figure 1), other things being equal, would [should] have caused a sharp increase of the airborne fraction” — Hansen et al., 2013

          Why did the models predict that they’d be coupled?

          http://onlinelibrary.wiley.com/doi/10.1029/2009GL040613/full
          “Despite the predictions of coupled climate-carbon cycle models, no trend in the airborne fraction can be found.”

          Sounds to me like you’re looking at the results, which don’t align with predictions, and claiming that that’s what you expected all along anyway. An ad hoc explanation. If they *had* aligned, I’m confident you’d say that *that’s* what you expected all along too.

          Ferdinand: “Between 1938 and 1950 the temperature dropped with 0.2°C, good for a drop of 3 ppmv CO2. Thus the emissions were good for an increase of 3 ppmv in that period. Human emissions in the same period were about 10 ppmv. Looks good to me…”

          In what way does this look good to you? If human emissions increased substantially, but the atmospheric concentration didn’t change, even declined somewhat in the early 1940s, that does not support your contention that nearly 100% of changes in atmospheric concentration are anthropogenic. Neither does the 1750 to 1875 period support that contention.

          And, as mentioned above, from 1970 to 2013, natural CO2 sources increased by 145.5 GtC/yr versus an increase of 4.6 GtC/yr for anthropogenic CO2 emissions. Why do you assume that the +145.5 GtC/yr increase in natural emissions had little to no effect on the atmospheric concentration, but only the +4.6 GtC/yr increase from anthropogenic sources did?

          1. Ferdinand Engelbeen

            Richard,

            I am not responsible for what James Hansen and the climate models expected. I never expected a fixed ratio, only was surprised that it was quite constant, until I saw the accumulated emissions – atmospheric CO2 curves:
            http://www.ferdinand-engelbeen.be/klimaat/klim_img/temp_emiss_increase.jpg
            Which show that emissions and increase are both slightly quadratic increasing, which gives a nice constant ratio between the two…

            My expectations are simply based on elementary physics: temperature influences the steady state level between ocean surface and atmosphere and any difference in the atmosphere (or the oceans) with the steady state is countered with a net flux proportional to the disturbance (Le Châtelier’s principle).

            Human emissions are quite steady increasing over time: slightly quadratic with only some leveling during times of economic crisis and very little year by year variability.
            CO2 sink rates only depend of the height of the extra CO2 pressure in the atmosphere (whatever the cause) over steady state, not of the momentary (human) emissions.

            That the net sink rate is directly proportional to the extra pressure in the atmosphere is easy to see in the evolution over time:
            extra pressure / net sink rate = e-fold decay time
            In 2012:
            110 ppmv / 2.15 ppmv/year = 51.2 years.
            The figures for 1988 (from Peter Dietze):
            60 ppmv, 1.13 ppmv/year, 53 years.
            In 1960:
            25 ppmv, 0.5 ppmv/year, 50 years.

            Seems quite linear to me with an e-fold decay rate of slightly over 50 years or a half life time of 30-35 years.

            The main point is that at the current rate of human emissions, the sinks can’t follow at the same pace and the difference accumulates in the atmosphere.
            That it is around 50% is pure coincidence (whatever the models expect) and mainly the effect of ever increasing human emissions. If human emissions would be constant, CO2 levels in the atmosphere would increase asymptotic towards a level where human emissions and sink rate are equal at a new steady state, thus with an “airborne fraction” of zero.

            That is elementary process dynamics…

            I never said that human CO2 is the only driver, temperature also has its (limited) role. In the period 1938-1950 the oceans cooled and without human emissions there would have been a drop of ~3 ppmv. Thus human emissions added 3 ppmv in the same period. Even since Mauna Loa, the yearly variability of the increase is 10-90% of human emissions and decadal it may be 40-60%. That is natural variability.

            Earlier periods are used by several researchers to “prove” that human civilization already expanded thousands of years ago, as CO2 and CH4 already increased slightly over equilibrium. In my opinion more speculation than science…

            “from 1970 to 2013, natural CO2 sources increased by 145.5 GtC/yr versus an increase of 4.6 GtC/yr for anthropogenic CO2 emissions.”

            I can’t find that reference back, but it is simply impossible: even if the natural sources increased with 145.5 GtC/year (because of what?), the natural sinks must have more than fully compensated for that increase, as the average net sink rate over that period did increase to 4.5 GtC/year.
            I suppose that you are mistaken here: the 145.5 GtC/year are the estimates of the rather constant natural emissions, not the increase in natural emissions over that period.

          2. Kenneth Richard

            KR: “from 1970 to 2013, natural CO2 sources increased by 145.5 GtC/yr versus an increase of 4.6 GtC/yr for anthropogenic CO2 emissions.”

            Ferdinand: “I can’t find that reference.”

            https://www.ipcc.ch/publications_and_data/ar4/wg1/en/ch7s7-3-1-3.html
            “Determined from atmospheric CO2 measurements (Keeling and Whorf, 2005, updated by S. Piper until 2006) at Mauna Loa (19°N) and South Pole (90°S) stations, consistent with the data shown in Figure 7.4, using a conversion factor of 2.12 GtC yr–1 = 1 ppm.”

            As described above…

            Using the IPCC conversion factor 2.12 GtC = 1 ppmv CO2 and the Mauna Loa CO2 ppm values for 1970 and 2013 (per NOAA.gov), we learn that the change, or net increase, in all CO2 carbon for 1970 to 2013 was +150.1 GtC (396.48 ppm [2013] – 325.68 ppm [1970] = 70.8 ppm x 2.12 GtC = 150.1 GtC).

            So again, 150.1 GtC was the total change in CO2 carbon increase from 1970 to 2013 using the IPCC formula. Of that increase, 4.6 GtC/yr was from anthropogenic emissions since 1970 (4.3 GtC/yr) to 2013 (8.9 GtC/yr).

            In total, then, the difference between 1970 natural emissions and 2013 natural emissions is +145.5 GtC/yr, which is 150.1 GtC/yr (total increase) – 4.6 GtC total increase from humans.

            Above, you claim that the “all the time” rate of natural carbon exchange is ~150 GtC/yr. For 2013, the IPCC puts the natural emission rate at 197.1 GtC/yr. Like the above, that also means that the change in yearly natural emissions is several times greater than the increase from anthropogenic sources if it is assumed that ~150 GtC/yr is the “all the time” rate and natural sources/sinks has reached ~200 GtC/yr for 2013.

            So again, where are all these additional natural sources and sinks coming from?

    2. Bart

      “Human emissions are twice as large…”

      So what? It has no bearing on the question of attribution. It is that awful, dumb-as-rocks pseudo-mass balance argument again.

  15. Kenneth Richard

    Ferdinand Engelbeen: “Willis is right … CO2 at the current mix makes about 20% of the greenhouse effect.”

    Willis said it was 41.134%, not 20%. So which one is he “right” about?

    1. Ferdinand Engelbeen

      Willis (and the absorption curves) said that it was over 40% if CO2 was alone, but less (about 20% in the literature) in the presence of water vapor, as both have overlapping bands where water vapor in most cases is already 100% absorbing and also a few bands where CO2 is 100% absorbing and water vapor is not or less…

      See the typical bands at:
      http://www.barrettbellamyclimate.com/page15.htm

  16. Ed Caryl

    At one time, I agreed with Salby that the oceans and rising temperature was the source of the rising CO2. Then I re-did the calculations using the actual tonnage of CO2 involved. It then became clear that the year-to-year variation (the delta) was due to ocean surface temperature, particularly in the tropics, but that about half the rise in CO2 was due to emissions. See the last article I did on the subject.
    https://notrickszone.com/2015/04/01/co2-emissions-and-ocean-flux-long-term-co2-increase-due-to-emissions-not-ocean-temperature/#sthash.GDrjB31E.dpbs

    1. Bart

      It’s almost all natural. Human inputs have little impact. Keep watching the rate of change. It’s going to get really interesting when La Nina kicks in.

      http://woodfortrees.org/plot/esrl-co2/derivative/mean:12/from:1979/plot/rss/offset:0.6/scale:0.22

    2. Ferdinand Engelbeen

      Ed, I have a long standing discussion with Bart, who believes that all CO2 increase is natural, because of the perfect match between two curves with an arbitrary factor and offset.

      No matter that conflicts with every single observation in the real world.

      No matter that you can show the same graph as a simple mix of the transient response of (mainly vegetation) CO2 to temperature variability and the residual of human emissions for the increased pressure in the atmosphere:

      http://www.ferdinand-engelbeen.be/klimaat/klim_img/had_co2_emiss_nat_deriv.jpg

      The response of the sinks to the increased pressure in the atmosphere still is in the middle of the variability, except for huge extremes:

      http://www.ferdinand-engelbeen.be/klimaat/klim_img/dco2_em6.jpg

      La Niña will align that again in the middle…

      1. Bart

        “…who believes that all CO2 increase is natural…”

        No, just the great majority of it, upwards of 80%.

        “No matter that conflicts with every single observation in the real world.”

        It conflicts with no observation, only with a narrative of observations.

        “…you can show the same graph as a simple mix…”

        A tortured rendering based on many assumptions and assertions lacking evidence, and passing off the remarkable consistency of the simple integral relationship as mere happenstance. Occam’s razor comes down squarely on my side.

        1. Ferdinand Engelbeen

          Bart for the 20th time:

          Your “solution” is physically and chemically impossible.

          – The equality of CO2 for the sinks, whatever its source, is violated if the natural circulation didn’t increase a fourfold over the past 55 years as human emissions and the increase in the atmosphere did. You have zero proof for any increase in natural carbon cycle.
          – The oxygen balance shows that vegetation is a net sink for CO2. The earth is greening. That excludes the biosphere as source of the extra CO2, it is a net, growing sink. You have zero proof of the opposite.
          – The 13C/12C ratio of the oceans (deep as well as surface) is higher than of the atmosphere. That excludes the oceans as main source of the increase in the atmosphere, as there is a firm decline in direct ratio to human emissions, as well as in the atmosphere, ocean surface and biosphere. You have zero proof of the presence of any other source of low-13C.
          -…

          Do you have even one single observation that shows that temperature is the main cause of the increase besides the “curve” (straigth lines) fitting you have done?

          For the calculations, my only assumptions are that the steady state level of the ocean surface follows Henry’s law and that the net sink rate is directly proportional to the ΔpCO2 between atmosphere and ocean surface at steady state. Simple, linear process dynamics. Maybe too simple for you?

          Temperature is mainly responsible for the variability of the CO2 rate of change, not for the bulk of the trend.

          No manipulation with factors and offsets to match two straight trends, as you did, whithout any physical base…

          1. Bart

            “The equality of CO2…”

            No, it isn’t. All that is required is that the natural process be dominant.

            “The oxygen balance… You have zero proof of the opposite.”

            As I am not claiming the opposite, I don’t need proof of it.

            ” The 13C/12C ratio of the oceans … You have zero proof of the presence of any other source of low-13C.”

            You have zero proof that fossil fuel combustion is the only possible explanation for the ratio being what it is. No matter how strongly you believe it, it is not proof. Moreover, as the diffusion process is very complex, it is not necessarily mutually exclusive to have the changing ratio be a result of fossil fuel combustion, yet still not have it be responsible for rising overall concentration.

            “Do you have even one single observation that shows that temperature is the main cause of the increase besides the “curve” (straigth lines) fitting you have done?”

            It only takes one fact to negate a theory. It is simply not possible (or, at least, exceedingly improbable) that the remarkable agreement between the trend and the variability in the dCO2/dt = k*(T-T0) relationship is mere happenstance, as you insist. The odds against it are astronomical.

            Against it, you have a tortured line of reasoning and a grab bag of various system responses which, when put together, resemble the data. But, it’s like fitting a curve with multiple parameters. Of course you can get a seeming fit that way, just as you can get a fit of planetary orbits using a geocentric model and epicycles. But, it’s meaningless. It requires too many items to line up just so, with no particular reason why they should.

            This is the basis of Occam’s razor – all things being equal, the simplest explanation is usually the right one. Because probabilities combine geometrically, and odds of a complicated model, with multiple assumptions that all have to be right, being the correct one consequently fall off in geometric fashion.

          2. Ferdinand Engelbeen

            Bart,

            The sinks don’t make any differentiation between human and natural CO2. I hope there we agree.
            The observations show a 4-fold increase in human emissions and a 4-fold increase of CO2 in the atmosphere. That means that the net sink rate also increased a 4-fold over the past 55 years.
            If the natural carbon cycle was dominant, it MUST have increased a 4-fold in lockstep with human emissions or you can’t have a 4-fold increase in net sink rate, or you violate the equality of all CO2 for the sinks…

            That is the case as well as for the increase in CO2 as for the decrease in δ13C.

            Further, your theory violates even the most elementary physical processes: if the CO2 partial pressure in the atmosphere increases, the outgassing at the upwelling places decreases and the uptake at the sink places increases. That completely lacks in your formula: a single temperature step emits a fixed amount of CO2 until eternity without any feedback from the increased CO2 pressure in the atmosphere…

            Even the nicest theory is busted if it violates any observation. Your theory violates every single observation on earth. No problem: your theory stands above observations (where have we heard that before?)…

            As proven before (a real proof, no alternative explanation possible): the variability in CO2 rate of change is from the response of (tropical) vegetation to fast, short temperature changes. As vegetation is a net absorber of CO2 on periods larger than 1-3 years, variability and slope are from different processes and there is no single factor that binds the amplitude and slope of the CO2 rate of change. Your single temperature controlled process is mathematically right, physically complete bogus. See:
            http://www.ferdinand-engelbeen.be/klimaat/klim_img/temp_dco2_d13C_mlo.jpg
            where the opposite changes in CO2 and δ13C prove that vegetation is the dominant response to temperature variability.

            About Occam’s razor: the simplest explanation that doesn’t violate any observation is usually the right one… As said before, the only assumptions made by me are that the sink rate is directly proportional to the extra pressure in the atmosphere over steady state and most of the variability is caused by the short term response of vegetation to temperature changes.

            That explains the T-CO2 ratio over the past 800,000 years from seasonal changes to glacial – interglacial periods.

          3. Bart

            “…if the CO2 partial pressure in the atmosphere increases, the outgassing at the upwelling places decreases and the uptake at the sink places increases.”

            But, more CO2 is upwelling all the time. It cannot be stopped. So, there is an accumulation in the surface oceans, which is going to force its way into an accumulation in the atmosphere.

            This is a dynamic process, Ferdinand. The pipeline into the surface system never stops pumping new CO2 in. If it is not gotten rid of, it will accumulate.

            “As proven before (a real proof, no alternative explanation possible): the variability in CO2 rate of change is from the response of (tropical) vegetation to fast, short temperature changes.”

            Nonsense. This isn’t proof. It’s rationalization of a preconceived conclusion. The trend and the years long variability are clearly from the same source.

        2. Kenneth Richard

          Ferdinand: “…who believes that all CO2 increase is natural…”

          Bart: “No, just the great majority of it, upwards of 80%.”

          Yes, this is exactly the straw-man tactic that Ferdinand uses that I was describing above. Someone writes that it’s “mostly” or “nearly all” natural, as Bart did, and someone like Ferdinand must purposely misrepresent that rather significant quantitative difference by claiming he wrote “all CO2 increase is natural.” By purposely re-writing what Bart actually wrote, Ferdinand can assert that his opponents have taken an extremist position… even though they have not.

          Even Dr. Salby himself has said that perhaps up to 30% of the CO2 increase could be anthropogenic.

          1. Ferdinand Engelbeen

            Kenneth,

            In my opinion, it makes little difference if someone says that 70% (Salby), 80% (Bart this time, near all at other occasions) or 97% (others) or 100% of the increase is natural, as good as the IPCC or near all scientists – including many skeptics – and I write that 90% or (near) all of the recent rise is human…

            In both cases the point is that either nature (temperature) is responsible or that humans are responsible for the bulk of the increase. Both viewpoints are “extreme” in the eyes of the opponent…

          2. Kenneth Richard

            In my opinion, it does make a difference when you misrepresent what someone has said by making up your own statement in an attempt to make it appear as extreme and marginal as possible.

            The difference between a 70 to 80% attribution and a 100% attribution is profound.

          3. yonason

            After digging facts out from under falsehoods, Lubos Motl concludes:

            “Decreasing pH of the oceans was called the ‘other problem with CO2’. We are finally beginning to learn that it is not really a problem. CO2 is life.” – Lubos Motl

            http://motls.blogspot.com/2008/04/oceans-love-co2-coccolithophores-say.html

          4. yonason

            2 years ago on NTZ
            https://notrickszone.com/2014/10/16/expert-blasts-alfred-wegener-institute-ocean-acidification-claim-clear-falsification-of-scientific-facts/#sthash.DlCO2M1u.dpbs

            They lies about “ocean acidification” have been know for a long time, and yet they persists among warmists, and those conned by them. And the fact that this is such a bald faced lie doesn’t seem to bother them.

            The REAL danger from Global Warming is not the warming, but those who pretend that, for a few measly trillion$ and total subjugation to their whims, they can do something about it.

    3. Ulric Lyons

      The warming of the AMO (Northern Atlantic) causes a large reduction in CO2 uptake, being one of the larger CO2 sinks on the planet.

      1. AndyG55

        AMO is over its peak, and looks like it is heading downwards.

        Let’s hope it doesn’t take too much aCO2 with it.

  17. yonason

    Write all the equations you want, but when push comes to shove, give me data any time. And the data say the pH ain’t a changin.
    http://www.sanctuarysimon.org/regional_docs/monitoring_projects/100240_167.pdf

    And STOP CALLING IT “ACIDIFICATION!” It’s basic, and if it becomes progressively less basic, it is not being “acidified.” Only when it goes below 7.0 does it become acidic, and only then can further decreases be called acidification.

    Perverting the terminology is a propaganda tool. It is NOT science.

    1. AndyG55

      Basically every river that has flowed into the oceans over millions of years, has been on the acidic side of neutral…

      .. yet ocean pH remains steadfastly around a pH of about 8.

      If anyone thinks a minor change in aCO2 concentration (some coming from the oceans) is going to affect ocean pH in any way whatsoever……

      ….. they have rocks for brains.

    2. Ferdinand Engelbeen

      yonason,

      The seawater intake of Monterey is not very representative for the oceans as a whole: there is frequent upwelling from the deep oceans, which shows a quite different pH (and lots more dissolved CO2) than surface waters…

      It is like looking at temperature trends taken over an asphalted parking lot: no skeptic will accept that as representative for global temperatures…

      1. yonason

        “It is like looking at temperature trends taken over an asphalted parking lot:” – Ferdinand Engelbeen

        Oh, do you mean like the way they take temps these days to artificially enhance the warming?
        http://www.surfacestations.org/odd_sites.htm
        Right!

        “…there is frequent upwelling from the deep oceans, which shows a quite different pH (and lots more dissolved CO2) than surface waters…”

        I thought we were talking about CO2 from the atmosphere, not from the deep oceans?

        I know that deeper colder water has more dissolved CO2. But the warmer surface water can’t dissolve as much. That’s the point. In fact, the bay should be the best place to measure an interaction between the atmosphere and the ocean surface. If you don’t see any effect there, and you don’t, then you’ve got nothing.

        And another issue you implicitly raise is that the pH (notoriously difficult to directly and accurately measure in ocean water) varies not only from location to location at the surface, but also depends on temperature, depth, time of day, time of year, etc., as well. Anyone who tells me they can detect an absolute change of a tenth of a pH unit in the oceans is either an idiot or a liar, so be careful who you believe on that, especially since the pH issue is just yet another warmist tempest in a teacup
        http://www.c3headlines.com/are-oceans-becoming-acidic/
        (don’t get distracted by the size of the teacup. It’s still just a contrived problem.)

        1. AndyG55

          “CO2 is not the enemy. Warmists are.”

          And luke-warmists.

        2. Ferdinand Engelbeen

          yonason,

          As good as you can’t use bad sited thermometers for global temperature trends, you can’t use the Monteray inlet pH for global pH trends. That is all I wanted to say.

          Upwelling and downwelling sites are only 5% each of the ocean surface. For most of the surface (the mixed layer), there is little exchange of CO2, nutrients, temperature,… with the deep oceans. That is where the “fast”est changes in pH are measurable with minimum disturbance of deep ocean upwelling.

          The total change in pH by all emissions since 1850 is calculated around 0.1 pH unit, by far no problem at all. But measurable in longer time series.
          Don’t underestimate current colorometric pH measuments: better than 0.001 pH unit. See:
          http://www.sciencedirect.com/science/article/pii/0967063793900488

          Besides that, pH can be accurately calculated from DIC (total inorganic carbon species) and TA (total alkalinity).

          1. yonason

            “For most of the surface (the mixed layer), there is little exchange of CO2, nutrients, temperature,… with the deep oceans. That is where the “fast”est changes in pH are measurable with minimum disturbance of deep ocean upwelling.” – Ferdinand Engelbeen

            Whoa Nelly!!!

            What do you mean “mixed layer,” is most of the surface, when when you say, in effect, it doesn’t mix?

            And how do you detect fractions of a tenth of a pH unit (yes, that’s what it would have to be) when pH variability is so naturally high, and pH detection is indirect and very complicated, and given that ocean pH is so highly variable?

            And just how do you think you are going to get enough data points over 70% of the earth’s surface to give you any meaningful numbers?

            The idea that you can detect such a tiny signal in so much noise is with so few measurements is, like all other warmist fantasies, utter nonsense!
            http://www.c3headlines.com/hysteria-polar-bearsseas-risingcoral-dying/

            If you have an hour and a half, this video is worth watching.
            https://www.youtube.com/watch?v=WwTmm1zcrJ0&index=4&list=PL9OovHFyFhwnNnUiMUsfgtttlMuYJFZCj

            Also see here
            https://climatism.wordpress.com/category/ocean-acidification/

            And, even if the pH were to fall, it wouldn’t be a problem
            http://hockeyschtick.blogspot.com/2013/08/new-paper-finds-ocean-acidification.html

          2. AndyG55

            “And how do you detect fractions of a tenth of a pH unit (yes, that’s what it would have to be) when pH variability is so naturally high, and pH detection is indirect and very complicated, and given that ocean pH is so highly variable?”

            WELL SAID..

            But Ferd falls for that sort of stuff. DOH !!

          3. AndyG55

            “The total change in pH by all emissions since 1850 is calculated around 0.1 pH unit, by far no problem at all. But measurable in longer time series.”

            Lets repeat that last line,

            “But measurable in longer time series.”

            What a load of utter BS !!!

          4. yonason

            “Don’t underestimate current colorometric pH measuments:” – Ferdinand Engelbeen

            Don’t overestimate them.

          5. Ferdinand Engelbeen

            yonason,

            If you make a vertical profile of the oceans, there is a huge difference between the upper (few) hundred meter(s) of the oceans and the rest of the oceans for pH, pCO2, nutrients, DIC, temperature, you name what else. That part of the oceans contains most of oceanic life, that is where photosynthesis works and a direct, fast (less than a year) exchange of CO2 and oxygen with the atmosphere. Besides the organic carbonate pump, there is very little vertical exchange between ocean surface and deep oceans, except for the limited upwelling and sink zones. The main – even so limited – mixing is horizontally.

            pH variability is high (so are tide gauges bery noisy too), but that doesn’t imply that no trend can be detected, even if it isn’t (yet) statistically significant. It will get significant when the time series gets long enough.
            For Bermuda local variability is +/- 0.05 unit, of which most is seasonal, the trend is already -0.04 units over ~30 years. If one ignores the seasonal component it is already significant.

            Proxies over longer periods are more problematic or need extreme fine accuracy (which they seldom have) to detect such small trends.

            Again, there is no harm to be expected from these small declines in pH, probably not even from much higher CO2 levels. But there is no harm either from expecting that human CO2 has a small influence on ocean surface pH levels…

          6. yonason

            “pH variability is high …, but that doesn’t imply that no trend can be detected, even if it isn’t (yet) statistically significant.” – Ferdinand Engelbeen

            If it can’t be shown to be statistically significant, it isn’t a real trend. And even if you find it is statistically significant, it may well still be unimportant.

            NEVER in the scientifically determined history of the world has either high temperature (22 DegC is the ceiling) or ocean “acidity” (with CO2 10 or more times higher than today) been a problem. There isn’t any reason to expect that they will be now or ever in the future, their paranoid delusions not withstanding.

            May clearer heads prevail
            http://motls.blogspot.com/2010/04/ocean-acidification-non-problem.html

            http://joannenova.com.au/2012/01/scripps-blockbuster-ocean-acidification-happens-all-the-time-naturally/

          7. Kenneth Richard

            “pH variability is high …, but that doesn’t imply that no trend can be detected, even if it isn’t (yet) statistically significant.” – Ferdinand Engelbeen

            Can you explain how you are able to detect a clear “trend” in an alleged -0.07 to -0.08 *total* U change in pH over the course of the last 200 years when the error bars are nearly as high as that alleged “trend” and you even acknowledge yourself that the “trend” isn’t statistically significant?

            You claim to be a skeptic, Ferdinand, but you are operating like a true believer in wholly accepting that you can find clear “trends” in data such as these. If it’s not even statistically significant, there’s no justifiable reason to claim there’s a trend … unless you just want to believe that there is anyway. And, obviously, that *is* what you want to believe. So, essentially, if you believe it’s true, it is. Who needs data when you’ve got your beliefs?

            https://www.researchgate.net/profile/Gangjian_Wei/publication/283946141_Decadal_variability_in_seawater_pH_in_the_West_Pacific_Evidence_from_coral_d_11_B_records/links/5653244708aefe619b191e48.pdf
            “We here estimate the OA rates from the two long (>150 years) annually resolved pH records from the northern SCS (this study) and the northern GBR [Wei et al., 2009], and the results indicate annual rates of -0.00039 +/- 0.00025 yr and -0.00034 +/- 0.00022 yr for the northern SCS [South China Sea] and the northern GBR [Great Barrier Reef], respectively. It is worth noting that the errors of these estimates are fairly large with RSD of 65% for that these two time-series do not show significant decreasing trend for pH. Despite of such large errors, estimated from these rates, the seawater pH has decreased by about 0.07–0.08 U over the past 200 years in these regions.”

            [Fig. 2, page 5, red font shows pH varying more in a span of a few years – 0.1 to 0.4 – than it has for the entirety of the last 200 years (0.07 to 0.08).]

          8. AndyG55

            What is the whole of ocean pH this week, Ferd..

            And what “should” it be?

          9. DirkH

            AndyG55 17. May 2016 at 5:02 AM | Permalink
            ““And how do you detect fractions of a tenth of a pH unit (yes, that’s what it would have to be) when pH variability is so naturally high, and pH detection is indirect and very complicated, and given that ocean pH is so highly variable?”
            WELL SAID..
            But Ferd falls for that sort of stuff. DOH !!”

            Please. Signals have components on different timescales, or frequencies. Daily or seasonal fluctuations can be filtered out. As long as the law of large numbers holds, i.e. the signal is linear in the range of interest, this can be done with linear low pass filters.
            So a weak but existing linear trend over long timescales *CAN* be determined.

            Ed has just done a very similar thing with his filtering of the seasonal Global Warming signal. He explained the technique he used. I don’t remember you criticizing him for his approach (and, his approach was indeed valid).

          10. yonason

            Sorry, DirkH, but in the absence of statistical significance (Which F.E. admitted to), you do not know if the trend you are looking at might be “real,” and even more important, even if “significant,” if it even matters.
            http://www.statpac.com/surveys/statistical-significance.htm

            Many researchers get very excited when they have discovered a “statistically significant” finding, without really understanding what it means. When a statistic is significant, it simply means that you are very sure that the statistic is reliable. It doesn’t mean the finding is important or that it has any decision-making utility.

            There might be a trend, but we won’t know until the statistics tell us. And with ocean pH, it doesn’t and won’t ever matter.

            As to the ocean “signal,” we can’t even measure the “noise” accurately, because there is no DIRECT measure of pH. It’s quite a complex and essentially indirect process, for which there’s not independent confirmation.

            Are you even aware of how involved measuring ocean pH is? Here’s a collection of the SOP’s for the necessary procedures and calculations.
            http://cdiac.ornl.gov/ftp/oceans/Handbook_2007/Guide_all_in_one.pdf
            Looks like a fun way to spend a summer internship.

          11. yonason

            “Daily or seasonal fluctuations can be filtered out.” – DirkH

            But what do you do when the noise IS the signal? And what do you “filter” out, when removal of the “noise” alters the signal?

          12. DirkH

            yonason 18. May 2016 at 12:59 AM | Permalink
            ““Daily or seasonal fluctuations can be filtered out.” – DirkH
            But what do you do when the noise IS the signal? And what do you “filter” out, when removal of the “noise” alters the signal?”

            When I look for a signal in the frequency range below 1 milliHz (for instance) I can safely suppress faster components. The signal below my cutoff frequency might be noise, might be periodic, might be a linear slope. You see that after the filtering. Such filtering is just a separation of components.

            The problem you allude to is the fact that in a chaotic system -which is by necessity nonlinear- energy is shifted from hi to low frequency bands and vice versa. (violating linear superposition principle)

            In such cases, the filtering would remove information vital to the understanding of the system. I can’t tell to which degree this applies to the pH problem. Looking at the low frequency component is an analysis that should be done anyway.

          13. yonason

            @DirkH

            Yeah, that sounds like what I was trying to get at. Not sure your low-frequency approach would yield a meaningful result, but not sure it wouldn’t either. Might be worth doing.

      2. yonason

        “Ocean acidification isn’t a problem: detailed measurements of pH, in a trip to colder places or higher depths, the ocean gets less alkaline (not acidic yet!). The doubling of CO2 changes pH by 0.1 which is much smaller than the natural fluctuations of the ocean.” – Lubos Motl
        http://motls.blogspot.com/2012/05/will-happer-co2-friend-or-foe.html

        CO2 is not the enemy. Warmists are.

  18. gallopingcamel

    Salby correctly points out that temperature has led [CO2] for at least 850,000 years. This can be explained in terms of Henry’s law.
    https://diggingintheclay.wordpress.com/2013/05/04/the-dog-that-did-not-bark/

    Macquarie University has an impressive website if you like eye candy:
    http://www.mq.edu.au/

    Reality is much uglier:
    https://diggingintheclay.wordpress.com/2014/12/16/countering-consensus-calculations/

    If you don’t have time to read the above link here is an excerpt:
    QUOTE
    Things are even worse in Australia as the treatment of Murray Salby shows. Here is what annoyed his superiors at Macquarie (Sydney) leading to his being fired without “due process” when he was attending a function in Europe. His return air ticket was voided so that he had to pay for his return to Australia: http://wattsupwiththat.com/2013/11/22/excerpts-from-salbys-slide-show/
    UNQUOTE

  19. Ferdinand Engelbeen

    All,

    As I am preparing for a nice roundtrip in China, where Internet access will be difficult or even impossible, You will miss (or not) my comments here which will cease for several weeks…

    Anyway, I enjoyed the discussions (I like a good fight now and then) and I hope you did too…

    1. yonason

      Have a safe and enjoyable trip.

    2. DirkH

      We did, Ferdinand. Good luck.

    3. Bart

      So, that’s why you let me have the last word. And, here I thought I had converted you. Well, have a good trip, anyway.

    4. Colorado Wellington

      Belatedly, I join in wishing you safe travels, Ferdinand.

  20. Richard111

    Glad I’m not a scientist… I’d go nuts!

    Spent several years working in desert regions and due to an interest in astronomy at the time often spent all night out on the sand. One thing I noticed; even though midday temperatures could reach the middle forties, night time temperature could fall to zero when the sky was clear (that is when I would be looking at the stars)!!

    With that point alone I am unable to believe in AGW.

    1. AndyG55

      Yep, the CO2 is still up there, but where the **** is that back radiative warming.

      Back radiative warming come from H2O, not CO2

      1. Richard111

        Since I’ve been back in the UK and following the AGW farce I’ve noticed that on a clear, fairly calm sunny day with humidity at around 80%, things warm up nicely. Good weather for gardening. If it is still clear and fairly calm at sunset I note temperature drops quite rapidly. Later in the evening if a cloud bank moves in overhead, fairly common here in Southwest Wales, the local temperature stops falling and begins to rise. In fact I’ve known it rise almost to the daytime temperature.
        Now that is ‘back radiation’ from physical water droplets allowing heat stored in the ground to rise to the surface again as the radiative level from the ground reduces.
        So from my experience no ‘back radiation’ from CO2 or H2O until clouds appeared.

      2. DirkH

        AndyG55 21. May 2016 at 11:25 AM | Permalink | Reply
        “Yep, the CO2 is still up there, but where the **** is that back radiative warming.”

        Planck temperature of the two CO2 absorption/emission bands peak at -80 centigrade (200K) and at about 330 centrigrade. I guess the latter is not even active to a noticeable amount because there is no 330 dentigrade hot surface emitting IR photons in that range that could be absorbed and backradiated.

        So CO2 will give you the wam and cozy feeling of mid(polar)night at Vostok(Antarctica). That’s how warm its radiation should feel.

        Clouds on the other hand radiate warmth like a stack of mineral water bottles in your freezer, at a cozy +4 deg C maybe. So, way warmer.

        1. DirkH

          So, while we associate IR with “heat” -because hot objects not jet glowing emit IR-, ordinary sunlight with a color temperature (or Planck temperature) of say 2700 K is WAY, WAY, WAY “hotter” or energy-intense per photon, explaining how sunlight immediately drives you to sweat while walking even on a cold day.

  21. The diminishing effect of CO2 concentration on world temperature | edmhdotme

    […] “The vast majority of that [greenhouse] warming is contributed by water vapor. Together with cloud, it accounts for 98% of the greenhouse effect.”  See [iv] […]

By continuing to use the site, you agree to the use of cookies. more information

The cookie settings on this website are set to "allow cookies" to give you the best browsing experience possible. If you continue to use this website without changing your cookie settings or you click "Accept" below then you are consenting to this. More information at our Data Privacy Policy

Close